Крок 3 - Медицина 2014 весна (буклет)

1 / 200
Хворий 67-ми років, в анамнезі 8 років ІХС, 4 роки тому Q-інфаркт міокарда. Раптово, після емоційного навантаження, виник напад серцебиття, знепритомнів. Об’єктивно: стан важкий, шкіра волога, бліда. Пульс та ЧСС не підраховуються. АТ-130/80 мм рт.ст. ЕКГ: ЧСС- 200/хв., QRS деформовані, розширені (0,14 с), дисоціація роботи передсердь та шлуночків. Який із засобів є препаратом першої черги у даного хворого? A 67-year-old patient with an 8-year history of coronary artery disease, a Q-myocardial infarction 4 years ago. Suddenly, after emotional stress, a palpitation attack occurred, he fainted. About objectively: the condition is severe, the skin is moist, pale. The pulse and heart rate are not counted. BP-130/80 mm Hg. ECG: heart rate- 200/min., QRS deformed, widened (0.14 s), dissociation of the work of the atria and ventricles. Which of the remedies is the first-line drug for this patient?

Лідокаїн Lidocaine

Верапаміл Verapamil

Дигоксин Digoxin

Строфантин Strophantin

Натрію аденозинтрифосфат Sodium adenosine triphosphate

2 / 200
У хворого 46-ти років, що лікується з приводу гострого абсцесу середньої частки правої легені виник біль у правій половині грудної клітки, задишка, стан різко погіршився. Об’єктивно: ЧД- 28/хв., ціаноз, асиметрія грудної клітки. Над правою легенею визначається притуплення перкуторного звуку в нижніх відділах та тимпаніт у верхніх відділах. Аускультативно: різко ослаблене везикулярне дихання. Яке найбільш імовірне ускладнення виникло у хворого? A 46-year-old patient being treated for an acute abscess of the middle lobe of the right lung developed pain in the right half of the chest, shortness of breath, and his condition worsened sharply. Ob' objectively: BH- 28/min., cyanosis, chest asymmetry. Dullness of percussion sound in the lower parts and tympanitis in the upper parts is determined over the right lung. Auscultatively: sharply weakened vesicular breathing. What is the most likely complication that the patient has developed?

Правобічний ексудативний плеврит Right-sided exudative pleurisy

Правобічний гемопневмоторакс Right-sided hemopneumothorax

Правобічний обмежений піопневмото-ракс Right-sided limited pyopneumoto-rax

Правобічний тотальний піопневмоторакс Right-sided total pyopneumothorax

Правобічна емпієма плеври Right-sided pleural empyema

3 / 200
Жінка 21-го року, вагітність 10 тижнів, надійшла до лікарні зі скаргами на блювання до 10 раз на добу. Відмічаються гіперсалі-вація, відсутність апетиту, розлади смаку та нюху, сухість шкіри та язика. АТ- 100/60 мм рт.ст., ЧСС- 92/хв. Живіт м’який, безболісний. Діурез знижений. В сечі: питома вага -1020, ацетон ++. Яка найбільш імовірна патологія обумовила таку картину? A 21-year-old woman, 10 weeks pregnant, came to the hospital with complaints of vomiting up to 10 times a day. Hypersalivation, lack of appetite, taste disturbances are noted and smell, dry skin and tongue. Blood pressure - 100/60 mm Hg, heart rate - 92/min. The abdomen is soft, painless. Diuresis is reduced. In the urine: specific gravity -1020, acetone ++. Which is the most likely pathology caused such a picture?

Блювання вагітних Pregnant vomiting

Гострий апендицит Acute appendicitis

Гострий пієлонефрит Acute pyelonephritis

Харчова токсикоінфекція Food poisoning

Гострий гастрит Acute gastritis

4 / 200
Вночі у хворого з’явилися задуха, кашель з рясним харкотинням рожевого кольору. З анамнезу: страждає на гіпертонічну хворобу, варикозне розширення вен обох кінцівок, 2 роки тому переніс інфаркт міокарда. Об’єктивно: неспокійний, положення ортопное, акроціаноз, ЧДР- 40/хв. АТ-220/110 мм рт.ст. У легенях - різнокаліберні середньо- і великопухирчасті хрипи. Який найбільш імовірний діагноз? At night, the patient developed shortness of breath, a cough with copious sputum of pink color. From the anamnesis: he suffers from hypertension, varicose veins of both extremities, 2 years ago he suffered myocardial infarction. Objectively: restless, orthopneic position, acrocyanosis, blood pressure - 40/min. BP-220/110 mm Hg. In the lungs - medium- and large-caliber rales. What is the most likely diagnosis?

Астматичний статус Asthmatic status

Тромбоемболія легеневої артерії Thromboembolism of the pulmonary artery

Гостра лівошлуночкова недостатність Acute left ventricular failure

Легенева кровотеча Pulmonary hemorrhage

Крупозна пневмонія Croup pneumonia

5 / 200
Хворий 33-х років надійшов до приймального відділення після суїцидальної спроби отруєння вихлопними газами. Рівень свідомості - кома II. Шкірна покриви сухі, яскраворожевого кольору, тахіпное 36/хв., АТ- 120/80 мм рт.ст., ЧСС- 128/хв. Діурез знижений. Вогнищевої симптоматики немає. Даному хворому найбільш показане: A 33-year-old patient was admitted to the emergency room after a suicidal attempt at exhaust gas poisoning. The level of consciousness is coma II. The skin is dry, bright pink in color, tachypnea 36/min ., blood pressure - 120/80 mm Hg, heart rate - 128/min. Diuresis is reduced. There are no focal symptoms. This patient is most indicated for:

Інфузія гіпертонічних розчинів Infusion of hypertonic solutions

Оксигенотерапія Oxygenotherapy

Дезінтоксикаційна терапія Detoxification therapy

Введення великих доз глюкокортикоїдів Introduction of large doses of glucocorticoids

Проведення гіпербаричної оксигенації Performing hyperbaric oxygenation

6 / 200
Хвора 73-х років хворіє на цукровий діабет другого типу, приймає манініл. Протягом двох тижнів приймає по 2 таблетки гіпотіазиду. Об’єктивно: свідомість сплутана, шкірні покриви сухі, теплі, тургор м’язів знижений, температура тіла - 38oC, Ps-120/хв., АТ- 60/40 мм рт.ст., дихання поверхневе, язик сухий, запах ацетону відсутній. Глюкоза крові - 34 ммоль/л, ацетон в сечі - слабкопозитивний, Na+ сироватки - 162 ммоль/л, K + - 3,0 ммоль/л. Який розчин показаний для застосування в першу чергу? A 73-year-old patient suffers from type 2 diabetes, takes Maninil. She takes 2 hypothiazide tablets for two weeks. Objectively: consciousness is confused, skin dry, warm, muscle turgor is reduced, body temperature - 38oC, Ps-120/min., blood pressure - 60/40 mm Hg, shallow breathing, dry tongue, no smell of acetone. Blood glucose - 34 mmol/l , acetone in urine - weakly positive, serum Na+ - 162 mmol/l, K + - 3.0 mmol/l. What solution is indicated for use in the first place?

Розчин Рінгера Ringer's solution

Поляризуюча суміш Polarizing mixture

Фізіологічний розчин NaCL Saline NaCL

5% розчин глюкози 5% glucose solution

Гіпотонічний розчин NaCL Hypotonic solution of NaCL

7 / 200
Постраждалий 46-ти років, виявлений на місці ДТП, скаржиться на біль у ділянці таза та правого стегна. Об’єктивно: загальмований, стогне, шкіра бліда, язик сухий. ЧДР- 28/хв. Праве стегно деформоване, набрякле, у верхній його третині патологічна рухливість, крепітація кісткових уламків. Великий крововилив, набряк та біль під час пальпації в правих пахвинній, здухвинній та сідничній ділянках. Ps- 112/хв, АТ- 80/45 мм рт.ст. Якому ступеню травматичного шоку відповідає стан потерпілого? A 46-year-old victim found at the scene of an accident complains of pain in the pelvis and right thigh. Objectively: slowed down, moaning, pale skin, tongue dry. ChDR- 28/min. The right thigh is deformed, swollen, in its upper third there is pathological mobility, crepitation of bone fragments. Large hemorrhage, swelling and pain during palpation in the right inguinal, pubic and buttock areas. Ps- 112/min, AT - 80/45 mm Hg What degree of traumatic shock does the victim's condition correspond to?

- -

I I

IV IV

III III

II II

8 / 200
Дитина 2-х років надійшла до інфекційного відділення зі скаргами на багаторазове невпинне блювання, профузні рідкі випорожнення. Об’єктивно: адинамічна, риси обличчя загострені, тургор тканин різко знижений, акроціаноз. Шкірні покриви бліді, холодні з 'мармуровим'малюнком. Дихання поверхневе, тони серця глухі, по-чащені. АТ- 55/35 мм рт.ст. Анурія. Який невідкладний стан у дитини? A 2-year-old child was admitted to the infectious department with complaints of repeated incessant vomiting, profuse liquid stools. Objectively: adynamic, facial features are sharp, tissue turgor is sharp decreased, acrocyanosis. The skin is pale, cold with a 'marble' pattern. Breathing is shallow, heart sounds are dull, frequent. Blood pressure - 55/35 mm Hg. Anuria. What is the child's urgent condition?

Гіповолемічний шок Hypovolemic shock

Гостра ниркова недостатність Acute renal failure

Гостра надниркова недостатність Acute adrenal insufficiency

Гостра судинна недостатність Acute vascular insufficiency

Гостра серцева недостатність Acute heart failure

9 / 200
Дитині 8-ми років був введений пеніцилін. Через декілька хвилин після ін’єкції у дитини з’явилися інспіраторна задишка, шумне дихання, набряклість кисті і обличчя, свербіж шкірних покривів. Який з перерахованих препаратів слід ввести у першу чергу? An 8-year-old child was injected with penicillin. A few minutes after the injection, the child developed inspiratory shortness of breath, noisy breathing, swelling of the hand and face, itching skin. Which of the listed drugs should be administered first?

Ілюконат кальцію Calcium Iluconate

Піпольфен Pipolfen

Еуфілін Euphilin

Мезатон Mesaton

Преднізолон Prednisone

10 / 200
У дівчини 15-ти років при тривалому перебуванні в задушливому приміщенні з’явилися скарги на запаморочення, слабкість. Різко зблідла, тіло вкрилося холодним потом, шкіра набула 'мармурового'малюнку, з’явилися ціаноз губ, сопорозна свідомість, часте поверхневе дихання, тахікардія. Тони серця послаблені, АТ- 50/10 мм рт.ст. Який найбільш вірогідний діагноз? A 15-year-old girl complained of dizziness and weakness after being in a stuffy room for a long time. She turned pale, her body became covered in cold sweat, her skin became 'marble 'picture, cyanosis of the lips, sedated consciousness, frequent shallow breathing, tachycardia appeared. Heart tones are weakened, blood pressure - 50/10 mm Hg. What is the most likely diagnosis?

Колапс Collapse

Задушно-ціанотичний напад Suffocation-cyanotic attack

Мігрень Migraine

- -

Іостра серцева недостатність Acute heart failure

11 / 200
У дівчини 20-ти років запідозрена недостатність наднирників (хвороба Аддісона). Що з нижче перерахованого може допомогти при підтвердженні діагнозу? A 20-year-old girl is suspected of having adrenal insufficiency (Addison's disease). Which of the following can help confirm the diagnosis?

Гіпокаліємія Hypokalemia

Гіперглікемія Hyperglycemia

Гіпотермія Hypothermia

Артеріальна гіпертензія Hypertension

Гіпонатріємія Hyponatremia

12 / 200
Студент 17-ти років скаржиться на сухий кашель, дряпання в горлі, підвищення температури тіла до 38,1oC. За порадою друга приймав ампіцилін. Через 2 дні після початку захворювання з’явилася висипка по всьому тілу, рожевого кольору, 1 см у діаметрі, дещо припіднята. Який НАЙМЕНШ вірогідний діагноз? A 17-year-old student complains of a dry cough, a scratchy throat, an increase in body temperature to 38.1oC. He took ampicillin on the advice of a friend. 2 days after the start disease, a rash appeared all over the body, pink in color, 1 cm in diameter, slightly raised. What is the LEAST likely diagnosis?

Менінгококцемія Meningococcemia

Кір Measles

- -

Медикаментозний дерматит Drug dermatitis

Мононуклеоз Mononucleosis

13 / 200
Хвора 37-ми років страждає на цукровий діабет, за добу вводить 64 ОД інсуліну. На ніч ввела 28 ОД інсуліну, з’явилися тремор, пітливість, потім розвинулося психомоторне збудження, агресивність, мова незв’яза-на. Об’єктивно: тремор, шкіра волога. Періостальні рефлекси підвищені. Позитивний симптом Бабинського. Тонічні та клонічні судоми. Який найбільш імовірний діагноз? A 37-year-old patient suffers from diabetes, injects 64 units of insulin per day. At night, she injected 28 units of insulin, tremors, sweating appeared, then developed psychomotor excitement, aggressiveness, incoherent speech. Objectively: tremor, moist skin. Periosteal reflexes are increased. Positive Babinsky symptom. Tonic and clonic convulsions. What is the most likely diagnosis?

Гіперосмолярна кома Hyperosmolar coma

Гіпоглікемічна кома Hypoglycemic coma

Лактацидемічна кома Lactacidemic coma

Діабетична кома Diabetic coma

Епілепсія Epilepsy

14 / 200
Після падіння та забою голови потерпілий 24-х років на декілька хвилин знепритомнів. Після повернення свідомості почувається досить добре, але скаржиться на помірний головний біль, є ретроградна амнезія, вогнищної неврологічної симптоматики нема. Який найбільш імовірний діагноз? After falling and hitting his head, the 24-year-old victim passed out for a few minutes. After regaining consciousness, he feels quite well, but complains of a moderate headache, has retrograde amnesia, there are no focal neurological symptoms. What is the most likely diagnosis?

Епілептичний напад Epileptic attack

Гостре порушення мозкового кровообігу Acute cerebrovascular accident

Зомління Grinding

Струс мозку Concussion

Забій мозку Brain stroke

15 / 200
Хвора 49-ти років доставлена каретою швидкої допомоги в ЛОР клініку із скаргами на відчуття здавлення в горлі, утруднене дихання, кашель, відчуття наявності стороннього тіла в горлі. Захворювання пов’язує з контактом із миючими хімічними засобами, з якими контактувала 3 години тому. Об’єктивно: слизова нижнього відділу глотки та гортані блідо-рожевого водянистого набряку, вхід в гортань звужений, дихання утруднене, стеноз першої стадії. Які дії в профілактиці асфіксії? A 49-year-old patient was taken by ambulance to the ENT clinic with complaints of a feeling of tightness in the throat, difficulty breathing, cough, feeling of a foreign body in the throat. Disease associated with contact with cleaning chemicals, with which she came in contact 3 hours ago. Objectively: mucous membrane of the lower part of the pharynx and larynx pale pink watery swelling, the entrance to the larynx is narrowed, breathing is difficult, stenosis of the first stage. What are the actions in the prevention of asphyxia ?

Протизапальна терапія, антибіотикотера-пія Anti-inflammatory therapy, antibiotic therapy

Інтубація Intubation

Бужування гортані Laryngitis

Десенсибілізуюча та протинабрякова терапія Desensitization and anti-edema therapy

Конікотомія Conicotomy

16 / 200
Хвора 25-ти років з метою знеболення прийняла внутрішньом’зево 2 мл 50% розчину анальгіну. На лівій половині обличчя навколо рота, на губах, слизовій оболонці ротової порожнини з’явився набряк. Набряк твердий, без ямок при натисканні. Через декілька годин стан хворої погіршився: з’явилась осиплість голосу, 'гавкаю-чий'кашель, утруднений вдих. Який попередній діагноз? A 25-year-old patient took 2 ml of a 50% analgin solution intramuscularly for pain relief. On the left side of the face around the mouth, on the lips, on the mucous membrane of the oral cavity swelling appeared. The swelling is firm, without dimples when pressed. After a few hours, the patient's condition worsened: a hoarseness of voice appeared, a barking cough, difficulty breathing. What was the previous diagnosis?

Набряк Квінке Quincke edema

Медикаментозна хвороба Medicine disease

Анафілактичний шок Anaphylactic shock

Токсикодермія Toxicoderma

Кропивниця (алергічна форма) Hives (allergic form)

17 / 200
У дівчинки 9-ти місяців виникла раптова зупинка дихання. Розпочато штучне дихання за методом 'рот до рота'. Що є показником правильності техніки проведення штучного дихання? A 9-month-old girl suddenly stopped breathing. Mouth-to-mouth artificial respiration was started. What is an indicator of the correctness of the artificial respiration technique?

Екскурсії грудної клітки хворого Excursions of the patient's chest

Зміна забарвлення шкіри Change in skin color

Дихальні шуми при аускультації легень Breath sounds during lung auscultation

Відновлення самостійного дихання Restoration of independent breathing

Зменшення проявів гіпоксії Reduction of manifestations of hypoxia

18 / 200
Хворий 34-х років перенесла крупозну пневмонію. На даний час скаржиться на озноб, задишку в спокої, загальну слабкість, головний 6іль, підвищення to до 39, 5oC, пітливість. Об’єктивно: зміщення назовні лівої межі серця, тони серця значно ослаблені, систолічний шум на верхівці, АТ- 80/50 мм рт.ст. У крові: лейк.- 10, 5 • 109/л, ',' A 34-year-old patient suffered from croup pneumonia. Currently, he complains of chills, shortness of breath at rest, general weakness, headache, an increase in temperature to 39.5oC, sweating. Objectively: displacement of the left border of the heart, heart sounds significantly weakened, systolic murmur at the apex, blood pressure - 80/50 mm Hg. In the blood: leuk. - 10.5 • 109/l, ','

Інфекційно-токсичний шок 6- Тромбоемболія легеневої артерії Infectious-toxic shock 6- Thromboembolism of the pulmonary artery

- 8%, ШЗЕ- 45 мм/год. У сечі: білок - 0,066 г/л, ер.-15-20 у п/з, гіалінові циліндри - 4-5 в п/з. Яке ускладнення виникло у хворого? - 8%, SZE- 45 mm/h. In urine: protein - 0.066 g/l, er.-15-20 in p/z, hyaline cylinders - 4-5 in p/z. What complication did the patient have?

Перикардит Pericarditis

Інфаркт міокарда Myocardial infarction

Кардіоміопатія Cardiomyopathy

19 / 200
У хворого 38-ми років через добу після вживання невідомої отрути з метою самогубства з’явились скарги на виражену слабкість, головний біль, нудоту, появу сечі кольору 'м’ясних помиїв', олігурія. Об’єктивно: АТ- 170/105 мм рт.ст., Ps- 84/хв., ритмічний. У сечі: білок - 0,99 г/л, еритроцити вкривають все поле зору, K + крові - 6,9 ммоль/л, креатинін -1,2 ммоль/л. Який найбільш імовірний діагноз? A 38-year-old patient complained of severe weakness, headache, nausea, and the appearance of 'm'-colored urine a day after consuming an unknown poison for the purpose of suicide 'clear slops', oliguria. Objectively: BP - 170/105 mm Hg, Ps - 84/min, rhythmic. In urine: protein - 0.99 g/l, erythrocytes cover the entire field of vision, K + blood - 6.9 mmol/l, creatinine -1.2 mmol/l. What is the most likely diagnosis?

Гостра ниркова недостатність Acute renal failure

Токсичний гастрит Toxic gastritis

Токсичний гепатит Toxic hepatitis

Токсична енцефалопатія Toxic encephalopathy

Гострий гломерулонефрит Acute glomerulonephritis

20 / 200
До реанімаційного відділення доставлена хвора 48-ми років без свідомості. Шкіра холодна, бліда, суха, акроціаноз. Щільний набряк обличчя, тіла, гомілок, температура тіла 33oC. Сухожильні рефлекси мляві. Ps-50/хв., малий, ритмічний. АТ- 90/60 мм рт.ст. Тони серця різко послаблені. Дихання рідке, глибоке. У крові: гіпохромна анемія, гіперхолестерінемія, високий рівень ТТГ крові. Який препарат необхідно призначити в першу чергу? A 48-year-old unconscious patient was brought to the intensive care unit. The skin is cold, pale, dry, acrocyanosis. Dense swelling of the face, body, legs, body temperature 33oC. Tendon reflexes are sluggish. Ps-50/min., small, rhythmic. BP- 90/60 mmHg. Heart tones are sharply weakened. Breathing is rare, deep. In the blood: hypochromic anemia, hypercholesterolemia, high level of blood TSH. What drug must be appointed first?

Преднізолон Prednisone

Мерказоліл Mercazolil

Анаприлін Anaprilin

Глюкоза Glucose

Трийодтиронін Triiodothyronine

21 / 200
Хворий 36-ти років знаходиться на лікуванні у відділенні реанімації з діагнозом: отруєння невідомою отрутою важкого ступеня. Кома. З анамнезу: знайдений вдома у непритомному стані. На АКТ головного мозку - без патології. Дані біохімічних досліджень крові: білірубін загальний - 120 мкмоль/л, глюкоза - 5 ммоль/л, ПТІ - 36%, загальний білок - 40 г/л, альбумін - 30%. Який найбільш імовірний діагноз? A 36-year-old patient is being treated in the intensive care unit with a diagnosis of severe poisoning by an unknown poison. Coma. From the history: found at home in an unconscious state. On CT brain - without pathology. Data of biochemical blood tests: total bilirubin - 120 μmol/l, glucose - 5 mmol/l, PTI - 36%, total protein - 40 g/l, albumin - 30%. What is the most likely diagnosis?

Отруєння дихлоретаном Dichloroethane poisoning

Отруєння ФОІ FOI poisoning

Отруєння серцевими глікозидами Cardiac glycoside poisoning

Отруєння атропіном Atropine poisoning

Отруєння протитуберкульозними препаратами Poisoning with antituberculosis drugs

22 / 200
З вогнища хімічного ураження, утвореного зарином, до МПП доставлено ураженого. Відзначаються напади ядухи за типом бронхіальної астми, фібрилярні посмикування жувальних м’язів. Який антидот необхідно ввести ураженому разом з атропіном на даному етапі медичної евакуації? From the focus of chemical damage caused by sarin, the affected person was brought to the MPP. There are attacks of dyspnoea according to the type of bronchial asthma, fibrillary twitching of the masticatory muscles. What antidote should be administered to the affected person along with atropine at this stage of medical evacuation?

Унітіол Unithiol

Дипіроксим Dipiroxime

Антиціан Antician

Тіосульфат натрію Sodium thiosulfate

Амілнітрит Amyl nitrite

23 / 200
Чоловік 74-х років викликав швидку допомогу з приводу блювання, здуття живота, наявності болючого грижового випинання у паху, яке не вправляється у черевну порожнину зі вчорашнього дня. Під час огляду лікарем швидкої допомоги грижове випинання вправилося, біль пройшов. Які дії лікаря? A 74-year-old man called an ambulance due to vomiting, abdominal distension, the presence of a painful hernial protrusion in the groin, which has not been inserted into the abdominal cavity since yesterday. Under during the examination by the emergency doctor, the hernial protrusion was relieved, the pain went away. What actions did the doctor take?

Рекомендувати хірургічне лікування у плановому порядку Recommend surgical treatment as planned

Призначити амбулаторне обстеження Schedule outpatient examination

Призначити дієту і холод на живіт Prescribe a diet and a cold on the stomach

Доставити хворого до стаціонару Deliver the patient to the hospital

Протягом години спостерігати за динамікою клінічної картини Over the course of an hour, observe the dynamics of the clinical picture

24 / 200
Хворий 29-ти років скаржиться на колючий біль у ділянці шиї справа, болючий акт ковтання, слинотечу. 2 години тому, під час їжі (їв рибу) відчув раптово болючість в правій половині горла. Який діагноз найбільш імовірний? A 29-year-old patient complains of stabbing pain in the neck area on the right, painful act of swallowing, drooling. 2 hours ago, while eating (eating fish), he suddenly felt soreness in the right half of the throat. What is the most likely diagnosis?

Заглотковий абсцес Pharyngeal abscess

Гострий тонзиліт Acute tonsillitis

Бокова киста шиї Lateral neck cyst

Паратонзилярний абсцес Paratonsillar abscess

Стороннє тіло гортаноглотки Laryngopharyngeal foreign body

25 / 200
В приймальний покій звернувся хворий із скаргами на носову кровотечу в помірній кількості. Із анамнезу виявлено, що кровотеча виникла раптово серед повного здоров’я. Яка тактика лікаря? A patient came to the waiting room with complaints of moderate nosebleeds. From the anamnesis, it was found that the bleeding occurred suddenly in the midst of complete health. What are the doctor's tactics?

Медикаментозна зупинка кровотечі Medical stoppage of bleeding

Задня тампонада Rear tamponade

Задня і передня тампонада Back and front tamponade

Перев’язка загальної сонної артерії Ligation of common carotid artery

Передня тампонада Anterior tamponade

26 / 200
Потерпілий був збитий автомашиною. У верхній третині лівого стегна рана 4х10 см, з якої виступає уламок кістки, значна варусна деформація стегна. Виберіть оптимальний метод іммобілізації: The victim was hit by a car. There is a 4x10 cm wound in the upper third of the left thigh, from which a bone fragment protrudes, significant varus deformation of the thigh. Choose the optimal method of immobilization:

Дві драбинчасті шини Two ladder tires

Шини Дітеріхса та Крамера Tires of Dieterichs and Kramer

Три драбинчасті шини Three ladder tires

Шина Дітеріхса Tire of Dieterichs

Шина Томаса Thomas Tire

27 / 200
У новонародженого хлопчика з вродженою вадою серця (тетрада Фалло) гостро розвинувся задушливо-ціанотичний напад. Який препарат слід ввести негайно разом з оксигенотерапією? A newborn boy with a congenital heart defect (tetrad of Fallot) acutely developed a suffocating-cyanotic attack. What drug should be administered immediately along with oxygen therapy?

Обзидан Obzydan

Сульфокамфокаїн Sulfocamphocaine

Кордіамін Cordiamine

Строфантин Strophantin

Допамін Dopamine

28 / 200
Після використання інсектициду у хворої 45-ти років з’явився головний біль, загальна слабкість, погіршення зору, напади задухи, болі в ділянці серця. Хвора у вимушеному положенні, плечовий пояс фіксований, шкіра та слизові ціанотичні, фібри-лярні посмикування м’язів обличчя, міоз. В легенях жорстке дихання, множинні сухі свистячі хрипи, брадикардія, зниження АТ до 90/60 мм рт.ст. Яка початкова доза введення 0,1% рну атропіну? After using an insecticide, a 45-year-old patient developed a headache, general weakness, impaired vision, attacks of suffocation, pain in the heart area. The patient is in a forced position , the shoulder girdle is fixed, the skin and mucous membranes are cyanotic, fibrillar twitching of the facial muscles, miosis. In the lungs, hard breathing, multiple dry whistling wheezing, bradycardia, a decrease in blood pressure to 90/60 mm Hg. What is the initial dose of administration 0, 1% of atropine?

4-8 мл 4-8 ml

6-8 мл 6-8 ml

1-2 мл 1-2 ml

8-10 мл 8-10 ml

2-4 мл 2-4 ml

29 / 200
Дівчинка 11-ти років скаржиться на кровотечу з піхви, що виникла на 5-й день менструації, слабкість, запаморочення. Кровотеча посилюється. У крові: Hb- 64 г/л, ер.-1, 8 • 109/л, тромб.- 280 • 109/л. Який найбільш імовірний діагноз? An 11-year-old girl complains of vaginal bleeding that occurred on the 5th day of menstruation, weakness, dizziness. Bleeding intensifies. In the blood: Hb- 64 g/l, er.-1, 8 • 109/l, thromb.- 280 • 109/l. What is the most probable diagnosis?

Геморагічний васкуліт Hemorrhagic vasculitis

Хвороба Віллібрандта Willibrandt's disease

Тромбоцитопенічна пурпура Thrombocytopenic purpura

Гіповітаміноз вітаміну К Vitamin K hypovitaminosis

Ювенільна маткова кровотеча Juvenile uterine bleeding

30 / 200
Чоловік 49-ти років з 30-ти років зловживає алкоголем. Толерантність, раніше висока, останнім часом впала, потяг до алкоголю компульсивний, запої по 14 днів, сформовано абстинентний синдром із блюванням, артеріальною гіпотензією. П’ять діб тому скінчився запій. На тлі безсоння, тремору кінцівок та гіпергідрозу три години тому став бачити ' гадюк'. Під час огляду лікарем швидкої допомоги дезорієнтований в часі, психомоторне збудження,' бачить зелену нитку'в порожній руці лікаря. Оберіть оптимальну схему медикаментозного лікування: A 49-year-old man has been abusing alcohol since he was 30 years old. Tolerance, previously high, has recently fallen, the desire for alcohol is compulsive, drinking for 14 days, formed abstinence syndrome with vomiting, arterial hypotension. Five days ago the binge ended. Against the background of insomnia, tremors of the limbs and hyperhidrosis, he began to see a 'viper' three hours ago. During the examination by an emergency doctor, he is disorientated in time, psychomotor agitation, 'sees a green thread' 'in the doctor's empty hand. Choose the optimal scheme of drug treatment:

Внутрішньовенно крапельно ввести 25 мг аміназину Enter 25 mg of aminazine intravenously

Внутрішньовенно ввести 20 мг седуксену Enter 20 mg of seduxen intravenously

Дати всередину 300 мг фенобарбіталу Give orally 300 mg of phenobarbital

Дати всередину 10 мг рудотелю Give inside 10 mg of rudotel

Внутрішньовенно ввести 25 мг тізерцину Enter 25 mg of tizercine intravenously

31 / 200
Хвора 19-ти років скаржиться на слабкий біль у горлі впродовж трьох місяців. Лікувалась самостійно. Об’єктивно: загальний стан задовільний, температура 36,6oC. Під час фарингоскопії на правому піднебінному мигдалику виявлена виразка з підритими краями, сальним дном. В підщелепній ділянці знайдено збільшений рухомий лімфатичний вузол. Рентгенографія грудної клітки без патологічних змін. Який найбільш імовірний діагноз? A 19-year-old patient complains of a slight pain in the throat for three months. She was treated independently. Objectively: the general condition is satisfactory, the temperature is 36.6oC. During pharyngoscopy revealed an ulcer with pitted edges and a fatty bottom on the right palatine tonsil. An enlarged mobile lymph node was found in the submandibular area. Chest X-ray showed no pathological changes. What is the most likely diagnosis?

Дифтерія глотки Pharyngeal diphtheria

Сифіліс глотки Syphilis of the pharynx

Виразково-плівчаста ангіна Ulcerous-membranous angina

Туберкульоз глотки Pharyngeal Tuberculosis

Рак мигдалика Tonsil cancer

32 / 200
Вагітна 21-го року в терміні 38-39 тижнів надійшла після серії судомних припадків. Рівень свідомості - кома. Відзначається виражений набряк нижніх кінцівок, одутлість обличчя. АТ-170/120 мм рт.ст., чСС- 120/хв. Дихання часте, поверхневе. У сечі - протеї-нурія. Який найбільш імовірний діагноз? The pregnant woman of the 21st year at 38-39 weeks arrived after a series of convulsive seizures. The level of consciousness is a coma. There is pronounced swelling of the lower extremities, puffiness of the face. AT- 170/120 mm Hg, heart rate - 120/min. Breathing is frequent, shallow. There is proteinuria in the urine. What is the most likely diagnosis?

Епілепсія Epilepsy

Прееклампсія Preeclampsia

Еклампсія Eclampsia

Отруєння невідомою речовиною Poisoning by an unknown substance

Гостре порушення мозкового кровообігу Acute cerebrovascular accident

33 / 200
У дитини 12-ти років скарги на частішання випорожнень до 2-3 разів на добу з домішками слизу та крові протягом 2-х місяців. Запідозрений неспецифічний виразковий коліт. Який метод має вирішальне значення в діагностиці цього захворювання? A 12-year-old child complains of frequent stools up to 2-3 times a day with impurities of mucus and blood for 2 months. Nonspecific ulcerative colitis is suspected. What method is crucial in the diagnosis of this disease?

Ендоскопія товстого кишечнику Colon endoscopy

Рентгеноскопія товстого кишечнику X-ray of the large intestine

Рентгенографія товстого кишечнику X-ray of the colon

Ехоскопія черевної порожнини Echoscopy of the abdominal cavity

Бактеріологічне дослідження калу Bacteriological examination of feces

34 / 200
У дівчинки 7-ми років важкий напад бронхіальної астми триває 7 годин, ефекту від призначення інгаляційних ^-агоністів та дексазону в/м не відзначається. При обстеженні - ознаки 'німих легень', пригнічення свідомості. Яке лікування треба призначити насамперед? A 7-year-old girl has a severe attack of bronchial asthma lasting 7 hours, the effect of the appointment of inhaled ^-agonists and dexazone IV is not noted. During the examination - signs 'silent lungs', depression of consciousness. What treatment should be prescribed first?

Підвищити дозу інгаляційних -агоністів Increase the dose of inhaled agonists

Призначити кортикостероїдні гормони внутрішньовенно Prescribe corticosteroid hormones intravenously

Призначити інфузійну терапію Prescribe infusion therapy

Штучна вентиляція легень Artificial lung ventilation

Призначити високі дози еуфіліну внутрішньовенно Prescribe high doses of Euphilin intravenously

35 / 200
Яка найбільш раціональна методика ева- куації сечі з сечового міхура при гострій затримці сечовипускання внаслідок гострого паренхіматозного простатиту? What is the most rational method of evacuating urine from the bladder in acute urinary retention due to acute parenchymal prostatitis?

Надлонна капілярна пункція сечового міхура Suprapalm capillary puncture of the bladder

Епіцистостомія Epicystostomy

Катетеризація сечового міхура металевим катетером Catheterization of the urinary bladder with a metal catheter

Катетеризація сечового міхура еластичним катетером Catheterization of the urinary bladder with an elastic catheter

Троакарна цистостомія Trocar cystostomy

36 / 200
Пацієнт 36-ти років хворіє на цукровий діабет, 1 тип. Скаржиться на болі в правій ступні, наявність ранової поверхні з гнійними виділеннями із нориці. Пульсація на магістральних артеріях збережена. Яке ускладнення діабету виникло? A 36-year-old patient suffers from type 1 diabetes. He complains of pain in the right foot, the presence of a wound surface with purulent discharge from a fistula. Pulsation on the main arteries preserved. What complication of diabetes occurred?

Тромбоз вен ступні Vein thrombosis of the foot

Трофічна виразка Tropical ulcer

Суха гангрена правої ступні Dry gangrene of the right foot

Волога гангрена правої ступні Wet gangrene of the right foot

Бешиха правої ступні Right foot rash

37 / 200
Дівчина 14-ти років з незрозумілих для батьків причин намагалась покінчити з собою ( подряпала собі ножем руку), після чого втекла з дому. На прийомі відмовляється пояснити свій стан, плаче, замикається в собі, закриває обличчя руками, не дає до себе доторкнутися, здригається при спробі наблизитись до неї. При наданні невідкладної допомоги виявлені фізичні пошкодження. Яка тактика лікаря? A 14-year-old girl, for reasons unknown to her parents, tried to commit suicide (she scratched her arm with a knife), after which she ran away from home. At the reception, she refuses to explain her condition , cries, withdraws into herself, covers her face with her hands, does not allow herself to be touched, shudders when trying to get close to her. During the provision of emergency care, physical injuries were found. What are the doctor's tactics?

Провести психотерапевтичну бесіду Conduct a psychotherapeutic interview

Госпіталізувати для всебічного обстеження та лікування Hospitalize for comprehensive examination and treatment

Призначити медикаментозне лікування Prescribe medical treatment

Відпустити додому Let go home

Забезпечити захист від насильства в домашніх умовах Provide protection against domestic violence

38 / 200
У потерпілого з отруєнням однією з сильних неорганічних кислот у місцях контакту кислоти зі слизовими оболонками виявлені шкурки жовтуватого кольору. Яка кислота найбільш імовірно стала причиною отруєння? A victim of poisoning with one of the strong inorganic acids has yellowish skins in the places of contact of the acid with the mucous membranes. Which acid most likely caused the poisoning?

Молочна Dairy

Сірчана Sulphur

Азотна Nitrogen

Хлоридна Chloride

Фосфорна Phosphorna

39 / 200
Хворий 53-х років скаржаться на біль у ділянці прямої кишки при дефекації, яскравочервону кров на калових масах, яка з’являється на початку акту дефекації, закрепи. Загальний стан задовільний. Тони серця ритмічні, ЧСС- 74/хв. Живіт м’який, не болючий. Яке інструментальне дослідження найбільш інформативне для встановлення остаточного діагнозу? A 53-year-old patient complains of pain in the region of the rectum during defecation, bright red blood in the stool, which appears at the beginning of the act of defecation, constipation. General the condition is satisfactory. The heart sounds are rhythmic, the heart rate is 74/min. The abdomen is soft, not painful. What instrumental examination is the most informative for establishing the final diagnosis?

Іригоскопія Irigoscopy

Пальцеве дослідження прямої кишки Digital rectal examination

Копрограма Coprogram

Ректоскопія Rectoscopy

Фіброколоноскопія Fibrocolonoscopy

40 / 200
До приймального відділення доставлено хворого 56-ти років, який упродовж 3-х років страждає на пахово-калиткову грижу. 10 годин тому грижа защемилась. Стан погіршувався, хворий викликав бригаду швидкої допомоги. Під час гігієнічної ванни грижа вправилась. Хворий відчув себе здоровим і проситься додому. Яка тактика? A 56-year-old patient, who has been suffering from an inguinal hernia for 3 years, was brought to the admission department. 10 hours ago, the hernia got pinched. The condition worsened, the patient called an ambulance. During a hygienic bath, the hernia healed. The patient felt healthy and asked to go home. What are the tactics?

Госпіталізувати хворого до хірургічного відділення, провести операцію в плановому порядку Hospitalize the patient to the surgical department, perform the operation as planned

Порекомендувати запобігати фізичному навантаженню і носити бандаж Recommend to prevent physical exertion and wear a bandage

Провести огляд та додаткові методи дослідження і відпустити хворого додому Conduct an examination and additional research methods and send the patient home

Терміново госпіталізувати хворого до хірургічного відділення та прооперувати Urgently hospitalize the patient to the surgical department and operate

При покращенні відпустити хворого з рекомендацією прооперуватись у плановому порядку In case of improvement, release the patient with a recommendation to operate as planned

41 / 200
У хворої 45-ти років з переломом стегна під час пункції підключичної вени з правого боку з’явився біль у правій половині грудної клітки, задуха, кашель. Об’єктивно: ціаноз обличчя та шиї, АТ- 100/60 мм рт.ст., ЧСС-114/хв. Перкуторно - тупість на боці ураження. Аускультативно - відсутність дихання. Рентгенологічно - відсутність легеневого малюнка. Яке ускладнення при виконанні маніпуляції виникло у хворої? A 45-year-old patient with a hip fracture developed pain in the right half of the chest, shortness of breath, cough during a puncture of the subclavian vein on the right side. objectively: cyanosis of the face and neck, blood pressure - 100/60 mm Hg, heart rate - 114/min. Percussion - dullness on the side of the lesion. Auscultation - absence of breathing. X-ray - absence of a lung pattern. What complication did the patient experience during the manipulation ?

Правобічний піопневмоторакс Right-sided pyopneumothorax

Правобічний гемопневмоторакс Right-sided hemopneumothorax

Правобічний пневмоторакс Right-sided pneumothorax

Правобічна емпієма плеври Right-sided pleural empyema

Правобічний гемоторакс Right-sided hemothorax

42 / 200
Постраждала 22-х років під час прогулянки в лісі була вжалена змією. Через 8 годин відмічається запаморочення, шкірні пориви бліді, холодні на дотик. Права рука набрякла та ціанотична від пальців до плеча. АТ- 70/40 мм рт.ст., Ps- 122 /хв., ЧД-22/хв. Яку невідкладну допомогу треба надати в першу чергу? A 22-year-old victim was bitten by a snake while walking in the forest. After 8 hours, dizziness is noted, the skin rashes are pale, cold to the touch. The right hand is swollen and cyanotic from the fingers to the shoulder. Blood pressure - 70/40 mm Hg, Ps - 122/min, ChD-22/min. What emergency aid should be provided first?

Гострий гемодіаліз Acute hemodialysis

Введення антитоксичної сироватки Injection of antitoxic serum

Переривчастий плазмаферез Intermittent plasmapheresis

Форсований діурез Forced diuresis

Інфузійна терапія Infusion therapy

43 / 200
У чоловіка 49-ти років через 3 дні після оброблення туші корови вимушеного забою на правій кисті з’явилося свербляча пляма, потім міхурець, на місці якого утворився безболісний карбункул діаметром до 3 см та набряк, який сягає ліктя. Температура до 37,8°C. Який діагноз у хворого? A 49-year-old man developed an itchy spot on his right hand 3 days after processing the carcass of a forced-slaughter cow, then a blister, on the site of which a painless carbuncle formed with a diameter of up to 3 cm and swelling that reaches the elbow. The temperature is up to 37.8°C. What is the patient's diagnosis?

Еризипелоїд Erysipeloid

Сибірка Anthrax

Флегмона Phlegmon

Карбункул Carbuncle

Бешиха Beshikha

44 / 200
До приймально-дiагностичного відділення доставлений хлопчик 8-ми років без свідомості. Зінці звужені, на світло не реагують. Об’єктивно: шкіра суха, бліда, холодна на дотик. Слизові яскраво-червоні. Дихання поверхневе, запах ацетону. Дитина хворіє на цукровий діабет 2 роки. З якого розчину слід розпочинати інфузійну терапію? An unconscious 8-year-old boy was brought to the reception and diagnostic department. The pupils are narrowed, they do not react to light. Objectively: the skin is dry, pale, cold on touch. Mucous membranes are bright red. Breathing is shallow, the smell of acetone. The child has been suffering from diabetes for 2 years. With which solution should infusion therapy be started?

0,9% NaCl 0.9% NaCl

5% глюкози 5% glucose

5% альбуміну 5% albumin

4% Na2CO 3 4% Na2CO 3

10% глюкози 10% glucose

45 / 200
До лікаря звернулася дитина 13-ти років, у якій гостро з’явилися сухість у роті, порушення зору та виражена м’язова слабкість. При огляді звертав увагу симетричний птоз повік, мідріаз, млявість реакції зіниць, зниження ковтального рефлексу. З анамнезу відомо, що дитина їла овочеві консерви. Яке захворювання можна запідозрити у дитини? A 13-year-old child came to the doctor with acute dry mouth, impaired vision, and pronounced muscle weakness. During the examination, a symmetrical ptosis of the eyelids, mydriasis, sluggish reaction of the pupils, decrease in the swallowing reflex. It is known from the anamnesis that the child ate canned vegetables. What disease can be suspected in the child?

Бульбарна форма поліомієліту Bulbar form of poliomyelitis

Енцефаліт Encephalitis

Дифтерійна нейропатія Diphtheria neuropathy

Пухлина мозку Brain tumor

Ботулізм Botulism

46 / 200
У дитини 9-ти років, яка хворіє на цукровий діабет, на тлі кишкової інфекції виникли і наростають неврологічні симптоми: порушення орієнтації, галюцинації, фокальні судоми. Наявні ознаки дегідратації III ступеню, дихальних порушень немає, запах ацетону відсутній. Який попередній діагноз? A 9-year-old child with diabetes developed and is increasing neurological symptoms against the background of an intestinal infection: disorientation, hallucinations, focal convulsions. Present signs dehydration of the III degree, there are no respiratory disorders, there is no smell of acetone. What is the previous diagnosis?

Енцефаліт Encephalitis

Діабетична кетоацидотична кома Diabetic ketoacidotic coma

Пухлина головного мозку Brain tumor

Гіперосмолярна кома Hyperosmolar coma

Гіпоглікемічна кома Hypoglycemic coma

47 / 200
Жінка 44-х років 3 роки тому перенесла операцію - струмектомію, після якої була призначена замісна терапія. Останній час препарати приймала нерегулярно, в результаті чого стан різко погіршився. В коматозному стані доставлена до лікарні. Об’єктивно: ЧСС40 /хв., АТ- 50/30 мм рт.ст., to-34,2oC. Який діагноз найбільш імовірний? A 44-year-old woman underwent an operation - strumectomy 3 years ago, after which replacement therapy was prescribed. Recently, she took drugs irregularly, as a result of which her condition worsened sharply. She was brought to the hospital in a comatose state. Objectively: heart rate 40/min, blood pressure 50/30 mmHg, to-34.2oC. What is the most likely diagnosis?

Гіперглікемічна кома Hyperglycemic coma

Гіпертиреоїдна кома Hyperthyroid coma

Гіпотиреоїдна кома Hypothyroid coma

Надниркова недостатність Adrenal insufficiency

Гіпоглікемічна кома Hypoglycemic coma

48 / 200
Хворий 64-х років скаржиться на сильний біль у правій гомілці розпираючого характеру, підвищення температури тіла до 38oC. Об’єктивно: шкіра гомілки набрякла, гіперемована, під час пальпації різко болісна у ділянці гомілкових м’язів. Захворювання прогресує протягом 5 діб. Який найбільш вірогідний діагноз? A 64-year-old patient complains of severe pain in the right shin of a distending nature, an increase in body temperature to 38oC. Objectively: the skin of the shin is swollen, hyperemic, during palpation is sharply painful in the area of ​​the calf muscles. The disease progresses over 5 days. What is the most likely diagnosis?

Бешихове запалення правої гомілки Beshich's inflammation of the right lower leg

Післятромбофлебітичний синдром Postthrombophlebitic syndrome

Облітеруючий атеросклероз судин правої нижньої кінцівки Obliterating atherosclerosis of vessels of the right lower limb

Гострий флеботромбоз глибоких вен правої гомілки Acute phlebothrombosis of the deep veins of the right leg

Тромбофлебіт підшкірних вен правої гомілки Thrombophlebitis of the subcutaneous veins of the right leg

49 / 200
У триденного хлопчика загальний стан тяжкий: наростають млявість, адинамія, м’язова гіпотонія, гіпорефлексія. У пологах тривалий безводний проміжок (14 годин). Шкіра сіро-бліда, акроціаноз. Тахі-пное. Над легенями вкорочення перкутор-ного звуку в нижніх відділах, вологі дрібно-міхурцеві крепітуючі хрипи. Тахікардія, тони серця ослаблені. Живіт здутий. Печінка +3,5 см. У крові: анемія, лейкоцитоз. З крові виділена культура золотавого стафілококу. Який найбільш імовірний діагноз? The general condition of a three-day-old boy is severe: lethargy, adynamia, muscle hypotonia, hyporeflexia are increasing. During childbirth, there is a long dry period (14 hours). The skin is gray-pale , acrocyanosis. Tachy-pnea. Over the lungs, shortening of the percussion sound in the lower parts, wet small-vesicular crepitus rales. Tachycardia, heart tones are weakened. Abdomen is distended. Liver +3.5 cm. In the blood: anemia, leukocytosis. From the blood A culture of Staphylococcus aureus was isolated. What is the most likely diagnosis?

Пізній неонатальний сепсис Late neonatal sepsis

Вроджений кардит Congenital Carditis

Вроджена пневмонія Congenital pneumonia

Ранній неонатальний сепсис Early neonatal sepsis

Гнійний менінгіт Suppurative meningitis

50 / 200
У новонародженого хлопчика після проведення операції замінного переливання цитратної крові з’явились неспокій, різкий крик, напади ціанозу, тоніко-клонічні судоми. Вміст натрію в сироватці крові 142 мекв/л, магнію 0,9 мекв/л, кальцію - 1,6 ммоль/л, калію - 4,8 мекв/л. Яке електролітне порушення виникло у дитини після трансфузії цитратної крові? A newborn boy developed restlessness, a sharp cry, attacks of cyanosis, and tonic-clonic convulsions. The sodium content in the blood serum was 142 meq /l, magnesium 0.9 meq/l, calcium - 1.6 mmol/l, potassium - 4.8 meq/l. What electrolyte disturbance did the child develop after transfusion of citrated blood?

Гіпонатріємія Hyponatremia

Гіперкальціємія Hypercalcemia

Гіпокаліємія Hypokalemia

Гіпокальціємія Hypocalcemia

Гіпермагніємія Hypermagnesemia

51 / 200
Хворий 24-х років доставлений в лікарню з кровотечею з різаної рани, яка триває упродовж 4-х годин. Об’єктивно: шкіра бліда, пов’язка на лівій нозі просякла кров’ю. Правий колінний суглоб деформований, рухи в ньому обмежені. Рідний брат хворого страждає на гемофілію. У крові: Hb- 42 г/л, час кровотечі за Дюком - 3 хв., час зсідання крові за Лі-Уайтом - 20 хв. Кров не згортається. Яка невідкладна допомога? A 24-year-old patient was brought to the hospital with bleeding from a cut wound that lasted for 4 hours. Objectively: the skin is pale, a bandage on the left leg is soaked with blood. The right knee joint is deformed, its movements are limited. The patient's brother suffers from hemophilia. In the blood: Hb - 42 g/l, bleeding time according to Duke - 3 minutes, blood clotting time according to Lee-White - 20 min. The blood does not clot. What is the first aid?

В/м введення кріопреципітату V/m introduction of cryoprecipitate

В/в струминне введення кріопреципітату IV injection of cryoprecipitate

В/в краплинне введення кріопреципітату IV drip introduction of cryoprecipitate

В/в введення епсилон-амінокапронової кислоти IV administration of epsilon-aminocaproic acid

Трансфузія тромбоконцентрату Transfusion of platelet concentrate

52 / 200
Під час виконання будівельних робіт вдома чоловік 62-х років ненароком упав, зачепивши уламок бетонної плити, який травмував та перетиснув йому праву гомілку, під час чого хворий знепритомнів. Через деякий час сусіди викликали бригаду швидкої допомоги. Опритомнівши, вказати час, який був без свідомості, постраждалий не міг. Які заходи повинні бути здійснені медиками на місці події з метою профілактики токсемії та ниркової недостатності? During construction work at home, a 62-year-old man accidentally fell, catching a fragment of a concrete slab, which injured and crushed his right shin, during which the patient fainted. After some time, the neighbors called an ambulance. After regaining consciousness, the victim could not specify the time he was unconscious. What measures should be taken by medics at the scene in order to prevent toxemia and kidney failure?

Вагосимпатична блокада Vagosympathetic blockade

Введення фуросеміду довенно Introduction of furosemide daily

Накладання джгута на стегно справа Applying a tourniquet on the right thigh

Профілактичні заходи можливі лише в стаціонарі Preventive measures are possible only in a hospital

Введення аскорбінової кислоти з 5% глюкозою довенно Introduction of ascorbic acid with 5% glucose daily

53 / 200
У хворого 32-х років після діагностики відкритого пневмотораксу з пакету перев’язочного індивідуального наклали оклю-зійну пов’язку. При огляді лікарем через 40 хвилин: стан важкий, хворий покритий холодним потом, дихання поверхневе, часте, малий частий пульс, виражений ціаноз шкіри і губ. Уражена частина грудної клітки розширена, при перкусії - тимпаніт. Яка первинна лікарська допомога? After the diagnosis of an open pneumothorax, an occlusive bandage was applied to a 32-year-old patient from an individual dressing package. When examined by a doctor 40 minutes later: the condition is serious , the patient is covered with cold sweat, breathing is shallow, frequent, a small frequent pulse, pronounced cyanosis of the skin and lips. The affected part of the chest is expanded, on percussion - tympanitis. What is the primary medical care?

Оксигенотерапія в лежачому положенні хворого Oxygenotherapy in the lying position of the patient

Плевральна пункція в VII-VIII міжребір’ї Pleural puncture in the VII-VIII intercostal space

Вагосимпатична блокада за Вишневським Vagosympathetic blockade according to Vishnevsky

Плевральна пункція у II-III міжребір’ї Pleural puncture in the II-III intercostal space

Заміна пов’язки, наркотичні анальгетики Change of bandage, narcotic analgesics

54 / 200
Грудна клітина потерпілої 38-ми років була притиснута бортом вантажівки до стіни. Скарги на загальну слабкість, задишку, біль і відчуття тяжкості в грудній клітці на стороні ураження. Об’єктивно: блідість шкіри, тахікардія, пульс слабкого наповнення, артеріальний тиск знижений, симптоми недостатності дихання. Вкажіть обсяг первинної лікарської допомоги: The chest of the 38-year-old victim was pressed against the wall by the side of the truck. Complaints of general weakness, shortness of breath, pain and a feeling of heaviness in the chest on the affected side. About Objectively: pale skin, tachycardia, weak pulse, low blood pressure, symptoms of respiratory failure. Specify the amount of primary medical care:

Негайна госпіталізація до травматологічного відділення Immediate hospitalization in the trauma department

Серцеві глікозиди, дихальні аналептики Cardiac glycosides, respiratory analeptics

Напівсидяче положення, анальгетики, інгаляція кисню Semi-sitting position, analgesics, oxygen inhalation

Катетеризація центральної вени, інфузій-на терапія Central venous catheterization, infusion therapy

Плевральна пункція у VII міжребір’ї Pleural puncture in the VII intercostal space

55 / 200
Вагітна 25-ти років у терміні 32 тижні страждає на міастенію. З’явились розлади дихання, тахікардія, психомоторне збудження, яке змінюється в’ялістю, парезом кишечнику та сфінктерів. Які першочергові засоби невідкладної допомоги? A 25-year-old pregnant woman at 32 weeks of age suffers from myasthenia gravis. Respiratory disorders, tachycardia, psychomotor excitement, which is replaced by lethargy, intestinal paresis and sphincters. What are the first aid measures?

Профілактика гіпоксії плоду Prevention of fetal hypoxia

Призначення прозерину Proserin appointment

Штучна вентиляція легень Artificial lung ventilation

Застосування міорелаксантів Use of muscle relaxants

Термінове розродження Urgent delivery

56 / 200
Дитина народилася в машині швидкої допомоги на шляху до пологового будин- ку. У новонародженого відсутнє спонтанне дихання після погладжування шкіри уздовж хребта. Які подальші дії? A baby was born in an ambulance on the way to the maternity hospital. The newborn is not breathing spontaneously after stroking the skin along the spine. What are the next steps?

Почати штучну вентиляцію легень Start CPR

Повторювати погладжування Repeat swipe

Дати кисень Give oxygen

Поплескати по п’яті Pat your heel

Поплескати по сідницях Slap on the buttocks

57 / 200
Постраждалий 46-ти років. Добу тому отримав травму верхньої третини стегна внаслідок падіння важкого предмету. Скарги на біль у ділянці травми. Об’єктивно: праве стегно в об’ємі +4 см у порівнянні з контрлатеральною кінцівкою. Передньо-медіальна поверхня правого стегна синюшна, шкірні покриви напружені, визначається флюктуація, під час пальпації помірна болісність. Який найбільш імовірний діагноз та тактика ведення хворого? The victim is 46 years old. A day ago, he was injured in the upper third of the thigh as a result of a heavy object falling. Complaints of pain in the area of ​​the injury. Objectively: the right thigh in the +4 cm in comparison with the contralateral limb. The anterior-medial surface of the right thigh is bluish, the skin is tense, fluctuation is determined, during palpation there is moderate pain. What is the most likely diagnosis and tactics of patient management?

Флегмона стегна. Госпіталізація у хірургічний стаціонар Phlegmon of the thigh. Hospitalization in a surgical hospital

Ненапружена гематома стегна. Госпіталізація у хірургічний стаціонар для оперативного лікування Non-tense hip hematoma. Hospitalization in a surgical hospital for operative treatment

Перелом стегнової кістки. ^мобілізація кінцівки. Транспортування у травмпункт Fracture of femur. Limb mobilization. Transportation to trauma center

Тромбоз стегнової вени. Компрес з маззю Вишневського. Амбулаторне лікування у хірурга Thrombosis of the femoral vein. Compress with Vishnevsky ointment. Outpatient treatment at the surgeon

Гематома правого стегна. Місцева гіпотермія. Стискаюча пов’язка. Антибіотикопрофілактика Right thigh hematoma. Local hypothermia. Compression bandage. Antibiotic prophylaxis

58 / 200
У студента 20-ти років раптово з’явилась висока температура (39, 5°C), біль голови, який іррадіює в шию та спину, блювання, загальна гіперестезія, світлобоязливість, менінгеальний симптомокомплекс, ураження окорухових нервів. Хворий швидко впав у коматозний стан. У крові: нейтрофільний лейкоцитоз, ШЗЕ- 28 мм/год. Лікворний тиск 370 мм вод.ст., ліквор мутний, 800 нейтрофілів у 1 мм3, білок 12 г/л, цукор -1,2 ммоль/л. Який найбільш імовірний діагноз? A 20-year-old student suddenly developed a high temperature (39.5°C), a headache radiating to the neck and back, vomiting, general hyperesthesia, photophobia, meningeal symptom complex, damage to the oculomotor nerves. The patient quickly fell into a comatose state. In the blood: neutrophilic leukocytosis, ESR - 28 mm/h. Cerebrospinal fluid pressure 370 mm Hg, cerebrospinal fluid cloudy, 800 neutrophils in 1 mm3, protein 12 g/l, sugar -1.2 mmol/l. What is the most likely diagnosis?

Субарахноїдальний крововилив Subarachnoid hemorrhage

Серозний менінгіт Serous meningitis

Гнійний менінгоенцефаліт Suppurative meningoencephalitis

Субдуральна гематома Subdural hematoma

Туберкульозний менінгіт Tuberculous meningitis

59 / 200
Після вживання м’яса свині, яка була зарізана у зв’язку з хворобою, яку лікували пеніциліном, у хворого на атопічну екзему розвинулись: сильний свербіж шкіри, набряк обличчя, уртикарні висипання по тулубу, верхнім кінцівкам. Який найбільш імовірний діагноз? After eating the meat of a pig that was slaughtered for a disease treated with penicillin, a patient with atopic eczema developed: severe skin itching, swelling face, urticarial rashes on the trunk, upper limbs. What is the most likely diagnosis?

Гостра екзема Acute eczema

Гостра кропив’янка Acute urticaria

Атопічний дерматит Atopic dermatitis

Харчова токсикодермія Food toxicoderma

Медикаментозна токсикодермія Medicinal toxicoderma

60 / 200
У хворого на шостий день після отримання невеличкої рани правої кисті при ро-6оті в полі підвищилась температура тіла до 39oC, з’явились ускладнене ковтання, утруднене відкриття рота, 'сардонічна посмішка'. Яке ускладнення виникло у хворого? On the sixth day after receiving a small wound on the right hand, the patient's body temperature rose to 39oC in the field, difficulty swallowing, difficulty opening the mouth, 'sardonic smile'. What complication did the patient have?

Газова гангрена Gas gangrene

Правець Tetanus

Флегмона кисті Phlegmon of the hand

Бешихове запалення кисті Beshich inflammation of the hand

Флеботромбоз судин кисті Phlebothrombosis of hand vessels

61 / 200
Жінка 37-ми років доставлена до приймального відділення після автокатастрофи. Виражена задишка, ЧД- 60/хв., дихальні шуми значно послаблені справа. Першим кроком в лікуванні повинно бути: A 37-year-old woman was brought to the emergency department after a car accident. She has pronounced shortness of breath, BH-60/min., breathing noises are significantly reduced on the right. The first step in treatment should be to be:

Трахеостомія Tracheostomy

Інфузійна терапія Infusion therapy

Виконання перикардіоцентезу Pericardiocentesis

Інтубація трахеї Tracheal intubation

Пункція правої плевральної порожнини Puncture of the right pleural cavity

62 / 200
У дитини 11-ти місяців спостерігається підвищення температури тіла до 38, 5oC, багаторазове блювання, часті рідкі випорожнення до 10-15 разів за добу. На 3-й день захворювання - шкіра бліда з мармуровим малюнком, риси обличчя загострені. Велике тім’ячко запале. Адинамія. Діурез помірно знижений. Чим обумовлена тяжкість захворювання? An 11-month-old child has an increase in body temperature up to 38.5oC, repeated vomiting, frequent loose stools up to 10-15 times a day. On the 3rd the day of the disease - the skin is pale with a marble pattern, the facial features are sharpened. The big head is inflamed. Adynamia. Diuresis is moderately reduced. What causes the severity of the disease?

Токсико-ексикоз Toxiko-exykosis

ДВЗ-синдром DVZ-syndrome

Гостра ниркова недостатність Acute renal failure

Парез кишечнику Intestinal paresis

Нейротоксикоз Neurotoxicosis

63 / 200
У пацієнта 14-ти років після контакту з несправним електроприладом відзначається загальмованість, наростає ціаноз, пульс на периферичних артеріях відсутній, при аускультації серця - різке послаблення тонів, тахікардія. Які невідкладні заходи на місці пригоди: In a 14-year-old patient, after contact with a faulty electrical device, inhibition is noted, cyanosis is increasing, there is no pulse on the peripheral arteries, during auscultation of the heart - a sharp weakening of tones, tachycardia. What are the immediate measures at the scene of the accident:

Проведення комплексу первинних реанімаційних заходів, за можливістю дефібриляція шлуночків Performance of a complex of primary resuscitation measures, possibly defibrillation of the ventricles

Штучне дихання рот до рота, непрямий масаж серця Mouth-to-mouth artificial respiration, indirect heart massage

Доставити до соматичного відділення Deliver to somatic department

Внутрішньом’язево сибазон та інгаляційно - зволожений кисень Intramuscular sibazone and inhalation - humidified oxygen

Доставити у відділення інтенсивної терапії Deliver to intensive care unit

64 / 200
Дитина 10-ти років доставлена в стаціонар через 1 годину після укусу змії (в ліву ногу) зі скаргами на пекучий біль у місці укусу, нудоту, блювання, задишку, серцебиття. Під час огляду: місце укусу набрякле, на шкірі геморагічний синдром. Який головний фактор невідкладної допомоги у даному випадку? A 10-year-old child was taken to the hospital 1 hour after a snake bite (on the left leg) with complaints of burning pain at the site of the bite, nausea, vomiting, shortness of breath , palpitations. During the examination: the bite site is swollen, hemorrhagic syndrome on the skin. What is the main factor of emergency care in this case?

Введення антикоагулянтів Introduction of anticoagulants

Проведення інфузійної терапії з форсуванням діурезу Infusion therapy with diuresis forcing

Накладення джгута вище місця укусу Applying a tourniquet above the bite site

Обколювання місця укусу розчином адреналіну 1:10000 Surrounding the bite site with a 1:10000 adrenaline solution

Дробне введення протизміїної сироватки Small injection of anti-serum

65 / 200
Жінка 28-ми років в ургентному порядку надійшла до гінекологічного стаціонару зі скаргами на сильні кров’яні виділення з піхви, біль внизу живота переймоподібного характеру. Пацієнтка перебувала на обліку в жіночій консультації з приводу вагітності строком 11 тижнів. Вищевказані симптоми виникли після психоемоціонального напруження вдома. Який діагноз можна встановити? A 28-year-old woman urgently came to the gynecological hospital with complaints of heavy bleeding from the vagina, pain in the lower abdomen of a cramp-like nature. The patient was registered in a women's consultation regarding an 11-week pregnancy. The above-mentioned symptoms occurred after psycho-emotional stress at home. What diagnosis can be established?

Апоплексія яєчника Ovarian apoplexy

Гострий цистит Acute cystitis

Позаматкова вагітність Ectopic pregnancy

Викидень, що розпочався Started Miscarriage

Перекрут кісти яєчника Ovarian cyst torsion

66 / 200
У роділлі звуження тазу І ступеня. Передбачувана вага плоду - 4100. Тривалість II періоду пологів - 40 хв. Перейми набувають судомний характер, болісні. Матка має форму пісочного годинника. Серцебиття плоду глухе, 110/хв. При вагінальному дослідженні відкриття маткового вічка повне. Голівка плоду рухома, над входом у малий таз. Плідного міхура немає. Яка тактика ведення пологів? In labor, pelvic narrowing of the 1st degree. The expected weight of the fetus is 4100. The duration of the 2nd period of labor is 40 minutes. The contractions become convulsive, painful. The uterus has the shape of an hourglass . Fetal heartbeat is dull, 110/min. During vaginal examination, the opening of the uterine cavity is complete. The fetal head is mobile, above the entrance to the small pelvis. There is no amniotic sac. What are the tactics of childbirth?

Поворот плоду на ніжку, витягнення плоду за ніжку Turning the fruit on the leg, pulling the fruit by the leg

Інгаляційний наркоз. Кесарів розтин Inhalation anesthesia. Caesarean section

Родопосилення окситоцином Oxytocin fertilization

Краніотомія Craniotomy

Накладання акушерських щипців Applying obstetric forceps

67 / 200
Впершенароджуюча знаходиться у другому періоді пологів. Під час огляду виявлено брадикардію плоду до 90-100/хв., що не вирівнюється після потуги та заходів, направлених на лікування гіпоксії плоду. При піхвовому дослідженні: голівка плоду знаходиться у вузькій частині порожнини малого тазу. Яка тактика ведення пологів? The first-time mother is in the second stage of labor. During the examination, fetal bradycardia up to 90-100/min was detected, which does not level off after effort and measures aimed at treating hypoxia of the fetus. During vaginal examination: the head of the fetus is in the narrow part of the pelvic cavity. What are the tactics of childbirth?

Кесарів розтин Caesarean section

Накласти акушерські щипці Apply obstetric forceps

Розпочати внутрішньовенне введення окситоцину з метою посилення пологової діяльності Start intravenous oxytocin to increase labor

Виконати перінеотомію Perine perineotomy

Продовжити лікування гіпоксії плоду Continue treatment of fetal hypoxia

68 / 200
Хворий 40-ка років страждає на хворобу Іценко-Кушинга протягом 3-х років, останні 5 місяців не приймав медикаментозних засобів. Після переохолодження гостро з’явився біль у животі, блювання, знизився артеріальний тиск до 80/40 мм рт.ст., Ps- 94/хв. В яких умовах потрібно провести обстеження та лікування хворого? A 40-year-old patient has been suffering from Itsenko-Cushing's disease for 3 years, has not taken any medication for the past 5 months. After hypothermia, acute pain appeared in abdomen, vomiting, decreased blood pressure to 80/40 mm Hg, Ps - 94/min. Under what conditions should the patient be examined and treated?

В неврологічному відділенні In the neurological department

В кардіологічному відділенні In the cardiology department

В хірургічному відділенні In the surgical department

Амбулаторне лікування Outpatient treatment

В ендокринологічному відділенні In the endocrinology department

69 / 200
Бригада швидкої медичної допомоги, що прибула на місце дорожньо-транспортної пригоди через 2 хвилини, констатувала у 5-річної дитини відсутність функції зовнішнього дихання та кровообігу, у зв’язку з чим було розпочато проведення первинних реанімаційних заходів відповідно з прийнятим в усьому світі 'правилом АВС'. У чому його сутність? The emergency medical team, which arrived at the scene of the traffic accident after 2 minutes, determined that the 5-year-old child had no function of external breathing and blood circulation, in therefore, primary resuscitation measures were started in accordance with the globally accepted 'ABS rule'. What is its essence?

У необхідності знання населенням, як алфавіту, правил первинної реанімації There is a need for the population to know, like the alphabet, the rules of primary resuscitation

'Правило АВС'регламентує перелік медичної документації та порядок її заповнення 'ABS rule' regulates the list of medical documentation and the order of its filling

'Правило АВС'ніякого відношення до реанімації не має 'Rule ABC' has nothing to do with resuscitation

У необхідності суворо дотримуватися певної послідовності реанімаційних заходів по аналогії з буквами алфавіту If necessary, strictly adhere to a certain sequence of resuscitation measures by analogy with the letters of the alphabet

У комплексному застосуванні трьох прийомів, перші букви назв яких скорочено виглядають як 'АВС' In the complex application of three techniques, the first letters of the names of which look like 'ABC'

70 / 200
Дівчинка 1-го року доставлена до лікарні з діагнозом: гостра ниркова недостатність, стадія олігоанурії. У хворої відзначається м’язова слабкість, аритмія, зміни на ЕКГ ( розширення інтервалів PQ та QRS, високі, вузькі та симетричні зубці T). Які біохімічні зміни викликають ці порушення? A 1st-year-old girl was brought to the hospital with a diagnosis of acute renal failure, stage of oligoanuria. The patient has muscle weakness, arrhythmia, changes on the ECG (extension PQ and QRS intervals, tall, narrow and symmetrical T-waves). What biochemical changes cause these disturbances?

Гіпокальціємія Hypocalcemia

Гіпокаліємія Hypokalemia

Ацидоз Acidosis

Гіперкальціємія Hypercalcemia

Гіперкаліємія Hyperkalemia

71 / 200
Вагітна 23-х років, яка знаходилась певний час у горизонтальному положенні на спині, знепритомніла. Діагностовано синдром нижньої порожнистої вени. Як у подальшому запобігти виникненню даного стану? A 23-year-old pregnant woman, who was in a horizontal position on her back for some time, fainted. Inferior vena cava syndrome was diagnosed. How can this condition be prevented in the future?

Здійснювати контроль артеріального тиску Check blood pressure

Призначити калорійне харчування Prescribe calorie intake

Призначити лікувальну фізкультуру Prescribe physical therapy

Виключити лежання вагітної на спині Exclude pregnant woman lying on her back

Призначити ліжковий режим Assign bed mode

72 / 200
У хворого 46-ти років діагностована гіпоглікемічна кома. Які лікарські засоби необхідно призначити хворому в першу чергу? A 46-year-old patient has been diagnosed with a hypoglycemic coma. What medications should be prescribed to the patient first?

Кортикостероїди Corticosteroids

Глюкоза Glucose

Адреналін Adrenaline

Глюкагон Glucagon

Ноотропи Nootropics

73 / 200
У чоловіка 36-ти років раптово після велопробігу з’явився різкий біль у попереко- вій ділянці зліва, який іррадіює в ліву пахвинну ділянку, супроводжується нудотою, блюванням. Хворий не може вибрати положення, яке б зменшувало біль. Симптом Пастернацького позитивний зліва, сечі мало, гематурія. Ps- 86/хв., АТ-130/80 мм рт.ст. Який невідкладний стан розвинувся у хворого? A 36-year-old man suddenly developed sharp pain in the lumbar region on the left after cycling, radiating to the left inguinal region, accompanied by nausea and vomiting. The patient cannot choose a position that would reduce the pain. Pasternacki's symptom is positive on the left, urine is scanty, hematuria. Ps- 86/min., BP-130/80 mmHg. What emergency condition has developed in the patient?

Гострий пієлонефрит Acute pyelonephritis

Поперековий радикуліт Lumbar sciatica

Розрив селезінки Rupture of the spleen

Гострий гломерулонефрит Acute glomerulonephritis

Лівобічна ниркова колька Left renal colic

74 / 200
Дитина 10-ти років захворіла гостро після повернення з місцевості, де є випадки кишкової інфекції. З’явились рідкі випорожнення до 20 разів за добу без болю в животі та тенезмів, блювання. Випорожнення поступово втратили каловий характер, стали безбарвними, нагадують рисовий відвар. Температура тіла в межах 35, 5 — 36oC. Яке захворювання слід запідозрити? A 10-year-old child became acutely ill after returning from an area where there are cases of intestinal infection. There were loose stools up to 20 times a day without abdominal pain and tenesmus, vomiting. Stools gradually lost their fecal character, became colorless, resembled rice broth. Body temperature in the range of 35.5 — 36oC. What disease should be suspected?

Сальмонельоз Salmonellosis

Дизентерія Dysentery

Холера Cholera

Ротавірусний гастроентерит Rotavirus gastroenteritis

Ешерихіоз Escherichia

75 / 200
Жінка 25-ти років помітила виділення яскравої крові з прямої кишки в кінці дефекації. Найбільш імовірною причиною кровотечі є: A 25-year-old woman noticed the discharge of bright blood from the rectum at the end of defecation. The most likely cause of the bleeding is:

Поліпоз товстої кишки Colon polyposis

Дивертикул Меккеля Meckel's diverticulum

Внутрішній геморой Internal hemorrhoids

Рак товстої кишки Colon cancer

Дивертикульоз товстої кишки Diverticulosis of the colon

76 / 200
У породіллі 25-ти років, яка має 0(1) групу крові та Rh(-) існує загроза розвитку гемолітичної хвороби у новонародженого. При огляді дитини на що потрібно звернути увагу в першу чергу? A 25-year-old woman in labor who has 0(1) blood group and Rh(-) is at risk of developing hemolytic disease in a newborn. When examining a child, what need to pay attention in the first place?

Колір шкіри і слизових оболонок Color of skin and mucous membranes

Частоту серцевих скорочень Heart rate

Наявність рефлексу смоктання Presence of sucking reflex

Рівень артеріального тиску Blood pressure level

Кількість дихань за хвилину Number of breaths per minute

77 / 200
Дитина 4-х місяців госпіталізована з приводу частого блювання, рідких випорожнень більше 10 разів за добу після введення прикорму. Шкіра та слизові оболонки сухі, велике тім’ячко запале, температура тіла 38,4oC, ЧСС- 158/хв., тони серця глухі, живіт здутий, рідкі сечовиділення, дефіцит маси тіла від початкової - 9%. Ht- 50%. У крови: K+- 3,4 ммоль/л; Na+- 152 ммоль/л. Який першочерговий етап лікування? A 4-month-old child was hospitalized due to frequent vomiting, liquid stools more than 10 times a day after the introduction of complementary foods. The skin and mucous membranes are dry, the large head is inflamed , body temperature 38.4oC, heart rate - 158/min., dull heart sounds, distended abdomen, liquid urination, body weight deficit from initial - 9%. Ht - 50%. In blood: K+ - 3.4 mmol/l; Na+- 152 mmol/l. What is the primary stage of treatment?

Регідратаційна терапія Rehydration therapy

Жарознижуюча терапія Antipyretic therapy

Корекція харчування Nutrition correction

Призначення пробіотиків Prescribing probiotics

Антибактеріальна терапія Antibacterial therapy

78 / 200
Першовагітна у терміні 34 тижні госпіталізована зі скаргами на головний біль і погіршення зору. AT- 170/120 мм рт.ст. Набряки кінцівок, передньої черевної стінки, обличчя. Протеїнурія, циліндри гіалінові та зернисті. Протягом тижня інтенсивного лікування стан вагітної погіршився. На фоні усіх зазначених явищ хвора почала втрачати зір. Висновок окуліста - початкове відшарування сітківки. Пологової діяльності немає. Яка подальша тактика? A first-time pregnant woman at 34 weeks was hospitalized with complaints of headache and visual impairment. AT- 170/120 mm Hg. Swelling of the extremities, anterior abdominal wall, face . Proteinuria, the cylinders are hyaline and granular. During the week of intensive treatment, the condition of the pregnant woman worsened. Against the background of all the above-mentioned phenomena, the patient began to lose her vision. The ophthalmologist's conclusion is the initial detachment of the retina. There is no labor activity. What are the further tactics?

Кесарський розтин Caesarean section

Вскрити плодовий міхур Open amniotic sac

Консервативна терапія Conservative therapy

Екстирпація матки Uterus extirpation

Пологозбудження Parturition

79 / 200
У постраждалого внаслідок дії іонізуючого випромінювання виникли симптоми первинної реакції гострої променевої хвороби. Госпіталізований до стаціонару через 2 доби. Які з показників периферичної крові мають найбільше діагностичне значення у цей період? As a result of the action of ionizing radiation, the victim developed symptoms of the primary reaction of acute radiation sickness. He was hospitalized after 2 days. Which of the indicators of peripheral blood have the greatest diagnostic value during this period ?

Нейтрофіли Neutrophils

ШЗЕ ШЭ

Тромбоцити Platelets

Лімфоцити Lymphocytes

Еритроцити Erythrocytes

80 / 200
Дитина поступила до лікарні з гострим постгеморагічним синдромом (група крові А(П), Rh+). Проведена гемотрансфузія групою крові А(П), Rh_, але на 4-ту добу розвинулася олігоанурія, погіршився стан. Креатинін плазми 320 мкмоль/л, сечовина плазми 26 ммоль/л. Про який стан у дитини можна думати? The child was admitted to the hospital with acute post-hemorrhagic syndrome (blood group A(P), Rh+). Hemotransfusion with blood group A(P), Rh_ was performed, but on 4 -on that day, oligoanuria developed, the condition worsened. Plasma creatinine 320 μmol/l, plasma urea 26 mmol/l. What kind of condition can you think about in the child?

Постгемотрансфузійна гостра ниркова недостатність Post-hemotransfusion acute renal failure

Анафілактичний шок, гостра ниркова недостатність Anaphylactic shock, acute renal failure

Постгіпоксична гостра ниркова недостатність Posthypoxic acute renal failure

Постгіповолемічна гостра ниркова недостатність Post-hypovolemic acute renal failure

Постгеморагічна гостра ниркова недостатність Posthemorrhagic acute renal failure

81 / 200
У чоловіка 39-ти років раптово з’явився біль у епігастральній ділянці 3 години тому. Ps60/хв. Живіт напружений, різко болючий у всіх відділах, позитивний симптом ЩоткінаБлюмберга. Який найбільш імовірний діагноз? A 39-year-old man suddenly developed pain in the epigastric area 3 hours ago. Ps60/min. The abdomen is tense, sharply painful in all departments, a positive symptom ShttkinBlumberg. What is the most likely diagnosis?

Гострий панкреатит Acute pancreatitis

Гострий апендицит Acute appendicitis

Гостра кишкова непрохідність Acute intestinal obstruction

Перфоративна гастродуоденальна виразка Perforative gastroduodenal ulcer

Гострий холецистит Acute cholecystitis

82 / 200
Хворий 50-ти років доставлений в лікарню зі скаргами на багаторазове блювання з домішками жовчі, затримку випорожнень та газів. Три роки тому з приводу закритої травми живота виконана резекція тонкої кишки. Живіт здутий. Визначається шум плескоту та позитивний симптом Грєкова. Який найбільш імовірний діагноз? A 50-year-old patient was brought to the hospital with complaints of repeated vomiting with bile impurities, delayed bowel movements and gas. Three years ago, due to a closed abdominal injury, a resection of the thin intestines. The abdomen is distended. There is a gurgling noise and a positive Grekov's sign. What is the most likely diagnosis?

Перфоративна виразка шлунку Perforative gastric ulcer

Пухлина товстої кишки Colon tumor

Гострий холецистит Acute cholecystitis

Гостра спайкова тонкокишкова непрохідність Acute adhesion small intestinal obstruction

Гострий апендицит Acute appendicitis

83 / 200
До приймального відділення доставлений хлопчик 9-ти років з діагнозом: уто-плення ( неповне, 'сухе'). Об’єктивно: дитина загальмована, бліда, ЧСС- 65/хв., АТ-90/45 мм рт.ст. На місці пригоди надана перша допомога. Які наступні дії лікаря? A 9-year-old boy was brought to the reception department with a diagnosis of drowning (incomplete, 'dry'). Objectively: the child is retarded, pale, heart rate - 65/min., BP-90/45 mm Hg. First aid was provided at the scene of the accident. What are the next actions of the doctor?

Введення анальгетиків Introduction of analgesics

Введення серцевих глікозидів Introduction of cardiac glycosides

Введення допаміну Injection of dopamine

Введення седативних препаратів Introduction of sedative drugs

Проведення оксигенотерапії Carrying out oxygen therapy

84 / 200
У дівчинки 11-ти років слабкість, часті рідкі випорожнення, багаторазове блювання, які з’явилися після початку кишкових розладів. Температура 36oC, шкіра землиста, суха, пульс прискорений, артеріальний тиск знижений. Живіт безболісний. Випорожнення у вигляді 'рисового відвару'. Який найбільш імовірний діагноз? An 11-year-old girl has weakness, frequent liquid stools, repeated vomiting, which appeared after the onset of intestinal disorders. Temperature 36oC, skin earthy, dry, pulse accelerated, blood pressure is reduced. Abdomen is painless. Stools in the form of 'rice broth'. What is the most likely diagnosis?

Сальмонельоз Salmonellosis

Харчова токсикоінфекція Food poisoning

Ротавірусний гастроентерит Rotavirus gastroenteritis

Холера Cholera

Неспецифічний виразковий коліт Nonspecific ulcerative colitis

85 / 200
Хвора 49-ти років скаржиться на судоми м’язів рук, які виникли через 2 тижні після струмектомії. Об’єктивно: загальний стан задовільний, Ps- 78/хв, АТ- 125/70 мм рт.ст. Патологічних змін з боку внутрішніх органів немає. Позитивні симптоми Хвостека і Трусо. Який препарат потрібно ввести для купування судом? A 49-year-old patient complains of hand muscle cramps that occurred 2 weeks after strumectomy. Objectively: the general condition is satisfactory, Ps-78/ min, blood pressure - 125/70 mm Hg. There are no pathological changes on the part of internal organs. Positive symptoms of Khvostek and Truso. What drug should be administered for convulsions?

Протисудомні препарати Anticonvulsants

Препарати калію Potassium drugs

Нейролептики Neuroleptics

Седативні Sedatives

Препарати кальцію Calcium preparations

86 / 200
Хвора 52-х років зі слабкістю синусо-вого вузла під час ходьби раптово впала. Черговим лікарем при огляді констатовано відсутність свідомості, пульсації на сонних артеріях та тонів серця. Зіниці вузькі, шкіра бліда з ціанотичним відтінком, рідке поверхневе дихання. З чого в даному випадку необхідно розпочинати реанімаційні заходи? A 52-year-old patient with weakness of the sinus node suddenly fell while walking. The doctor on duty during the examination determined the absence of consciousness, pulsation on the carotid arteries and heart sounds. The pupils are narrow, the skin is pale with a cyanotic shade, shallow breathing. What should resuscitation measures be started in this case?

Удар кулаком по груднині A punch to the sternum

Внутрішньовенне введення ізоптину Intravenous Isoptin

Імплантація штучного водія ритму Implantation of an artificial pacemaker

Інтубація та проведення штучної вентиляції легень Intubation and artificial ventilation

Внутрішньовенне введення лазиксу Intravenous Lasix

87 / 200
У хворого 27-ми років після перелому стегна виникли задишка, легке запаморочення, серцебиття. Об’єктивно: ціаноз губ, тахіпное, тахікардія, акцент II тону над легеневою артерією. Лікар припустив наявність жирової емболії. ЕКГ у нормі. Якою має бути тактика лікаря? A 27-year-old patient developed shortness of breath, light dizziness, and palpitations after a hip fracture. Objectively: cyanosis of the lips, tachypnea, tachycardia, accentuation of the II tone above the pulmonary artery. The doctor assumed the presence of a fat embolism. The ECG is normal. What should be the doctor's tactics?

Лікувати амбулаторно Treat on an outpatient basis

Лікувати амбулаторно, госпіталізувати при погіршенні ЕКГ-показників Treat on an outpatient basis, hospitalize if ECG indicators deteriorate

Забезпечити консультацію кардіолога в продовж доби Ensure consultation of a cardiologist during the day

Негайно госпіталізувати хворого Immediately hospitalize the patient

Лікувати амбулаторно, спостерігати за хворим сумісно із кардіологом Treat on an outpatient basis, observe the patient together with a cardiologist

88 / 200
Жінка 67-ми років, яка 4 місяця тому назад перенесла інфаркт міокарда, поступила до приймального відділення. Об’єктивно: без свідомості, шкірні покрови та видимі слизові різко бліді, відсутність серцевої діяльності та дихання. На ЕКГ - фібриляція шлуночків. Як відновити ритм? A 67-year-old woman who suffered a myocardial infarction 4 months ago was admitted to the emergency department. Objectively: unconscious, the skin and visible mucous membranes are sharply pale, lack of heart activity and breathing. On the ECG - ventricular fibrillation. How to restore the rhythm?

Ввести глюкокортикоїди Enter glucocorticoids

ШВЛ через маску Mask ventilation

Ввести адреналін Inject adrenaline

Екстрена дефібриляція Emergency Defibrillation

Відкритий масаж серця Open heart massage

89 / 200
До відділення реанімації поступила дитина 7-ми років зі скаргами на підвищення температури до 38,3°C, почервоніння шкіри, які виникли вранці після вживання їжі та вітамінів (зі слів матері). Об’єктивно: шкірні покриви вкриті поліморфною висипкою у вигляді папул, везикул, геморагій, іноді є місця ерозій; симптом Нікольського позитивний. Такі ж прояви на слизових оболонках. Аускультативно: дихання жорсткого характеру, симетричне, провідні хрипи. ЧСС- 144/хв., тони серця ритмічні, глухі, систолічний шум на верхівці. На ЕКГ: метаболічні порушення міокарда. Який попередній діагноз? A 7-year-old child was admitted to the intensive care unit with complaints of an increase in temperature to 38.3°C, redness of the skin, which occurred in the morning after eating food and vitamins ( from the words of the mother). Objectively: the skin is covered with a polymorphic rash in the form of papules, vesicles, hemorrhages, sometimes there are places of erosion; Nikolsky's symptom is positive. The same manifestations on the mucous membranes. Auscultation: breathing of a hard nature, symmetrical, conducting wheezes. Heart rate - 144/min., heart sounds are rhythmic, dull, systolic murmur at the apex. On the ECG: metabolic disorders of the myocardium. What is the previous diagnosis?

Кір Measles

Поліморфна еритема (синдром Лайєлла ) Erythema polymorphic (Lyell's syndrome)

Скарлатина Scarlatina

Раптова екзантема (герпетична інфекція) Sudden rash (herpetic infection)

Краснуха Krasnukha

90 / 200
До відділення реанімації поступила дівчинка 5-ти місяців зі скаргами на наявність судом. Об’єктивно: температура 36,6°C, шкірні покриви блідо-рожевого кольору, теплі на дотик, вологі. Голова деформована, потилиця плоска, без волосся, лобні горби. Аускультативно: дихання пуерильне, симетричне; ритм правильний, тони гучні, ЧСС-134/хв. У крові: гіпохромна анемія I ступеню, анізоцитоз, пойкілоцитоз, гіпопротеї-немія, Na- 145, K- 4,2, Ca- 1,1. У відділенні реанімації судоми повторилися, з’явилися задишка інспіраторного характеру, сиплий голос, гавкаючий кашель, апное. Діагностовано рахіт, гіпокальціємія. Яка тактика лікаря? A 5-month-old girl was admitted to the intensive care unit with complaints of convulsions. Objectively: temperature 36.6°C, pale pink skin, warm to the touch, moist. The head is deformed, the back of the head is flat, without hair, frontal humps. Auscultation: breathing is puerile, symmetrical; the rhythm is correct, loud tones, HR-134/min. In the blood: hypochromic anemia of the 1st degree, anisocytosis, poikilocytosis, hypoproteinemia -nemia, Na- 145, K- 4.2, Ca- 1.1. In the intensive care unit, convulsions recurred, inspiratory dyspnea, hoarse voice, barking cough, apnea appeared. Rickets, hypocalcemia were diagnosed. What are the doctor's tactics?

10% кальцій хлорид 10% calcium chloride

ГОМК ГОМК

Вітамін D Vitamin D

Фенобарбітал Phenobarbital

Сибазон Sibazone

91 / 200
Вагітна 24-х років у терміні 40 тижнів надійшла до клініки зі скаргами на головний біль, нудоту, набряки ніг та передньої черевної стінки. АТ- 150/90 мм рт.ст. та 160/85 мм рт.ст., Ps- 92/хв. Серцебиття плоду 143/хв. Який метод дослідження необхідно зробити, щоб підтвердити діагноз прееклампсії? A 24-year-old pregnant woman at 40 weeks came to the clinic with complaints of headache, nausea, swelling of the legs and anterior abdominal wall. Blood pressure - 150/90 mm Hg and 160/85 mm Hg, Ps- 92/min. Fetal heartbeat 143/min. What research method should be done to confirm the diagnosis of preeclampsia?

Загальний аналіз крові General blood test

Біохімічний аналіз крові Biochemical blood analysis

Аналіз сечі на білок Urine analysis for protein

Коагулограма Coagulogram

Бактеріологічний аналіз сечі Urine bacteriological analysis

92 / 200
У хворої 24-х років захворювання почалося гостро з появи нудоти, багаторазового блювання, ознобу, болю у верхніх відділах живота. Практично одночасно з’явилися рідкі водянисті випорожнення без патологічних домішок 2 рази. Об’єктивно: стан середньої важкості, температура 37,6°C, Ps-86/хв., АТ-100/60 мм рт.ст., шкіра бліда, тургор збережений, язик незначно обкладений білим нальотом, живіт м’який, чутливий в епігастрії. Яку терапію слід провести у першу чергу? In a 24-year-old patient, the disease began acutely with the onset of nausea, repeated vomiting, chills, pain in the upper abdomen. Almost simultaneously, liquid watery stools appeared without pathological impurities 2 times. Objectively: the condition is of medium severity, temperature 37.6°C, Ps-86/min., BP-100/60 mm Hg, skin is pale, turgor is preserved, the tongue is slightly covered with a white coating, the abdomen is soft, sensitive in the epigastrium. What therapy should be carried out first?

Лоперамід Loperamide

5% розчин глюкози 5% glucose solution

Призначення сольового послаблюючого Prescription of saline laxative

Промивання шлунку Gastric lavage

Регідрон Regidron

93 / 200
В одному з сільських господарств проводилося оброблення сільськогосподарських рослин фосфорорганічними речовинами (карбофос). У помічника тракториста, який був без респіратора, через годину після роботи з’явилися головний біль, нудота, пітливість, слинотеча, порушення зору, відчуття страху. Яка антидотна терапія показана хворому? In one of the farms, agricultural plants were treated with organophosphorus substances (karbofos). The tractor driver's assistant, who was without a respirator, developed a headache an hour after work , nausea, sweating, salivation, impaired vision, feeling of fear. What antidote therapy is indicated for the patient?

Введення розчину хромосмону - 50 мл в/в, 5% розчину унітіолу - 5 мл в/в Introduction of chromosmon solution - 50 ml IV, 5% unitiol solution - 5 ml IV

Введення 40% розчину глюкози з 5% розчином вітаміну С - 5 мл в/в, 10% розчину кофеїну бензоату -1 мл п/ш Introduction of 40% glucose solution with 5% vitamin C solution - 5 ml IV, 10% caffeine benzoate solution - 1 ml IV

Введення 0,1% розчину атропіну - 2 мл в/в, 15% розчину діпіроксиму -1 мл в/м Introduction of 0.1% atropine solution - 2 ml IV, 15% dipiroxime solution - 1 ml IV

Введення 5% розчину унітіолу - 5 мл в/в, 20% розчину антиціану - 1 мл в/м Introduction of 5% unitiol solution - 5 ml IV, 20% solution of anticyanate - 1 ml IV

Введення 20% розчину антиціану - 1 мл в/м, вдихання амілнітриту через зволожену вату Introduction of 20% anticyanide solution - 1 ml in/m, inhalation of amyl nitrite through moistened cotton wool

94 / 200
У хворої 33-х років під час прийому їжі почався різкий переймоподібний кашель, посилилась задишка. Під час огляду шкіра бліда, акроціаноз, ЧД- 30/хв. При аускультації вислуховується хлопок над дихальними шляхами, дихання стридорозне. Яке дослідження найбільш доцільне при диференцій-ній діагностиці цього стану? A 33-year-old patient developed a sharp paroxysmal cough while eating, shortness of breath increased. During the examination, the skin is pale, acrocyanosis, BH-30/min. on auscultation, a clap is heard over the respiratory tract, stridorous breathing. What research is most appropriate in the differential diagnosis of this condition?

Оглядова рентгенографія органів грудної порожнини Surveillance X-ray of chest cavity

Комп’ютерна томографія органів грудної порожнини Computer tomography of the chest cavity

Фібробронхоскопія Fibrobronchoscopy

Спірографія Spirography

Бронхографія Bronchography

95 / 200
У хворого 62-х років діагностовано ішемічний інсульт. Прийнято рішення про початок тромболізисної терапії. В які строки від початку захворювання терапія буде максимально ефективною? A 62-year-old patient was diagnosed with an ischemic stroke. A decision was made to start thrombolysis therapy. At what time from the onset of the disease will the therapy be maximally effective?

До 3-х годин Up to 3 hours

До 1 доби Up to 1 day

До 6-ти годин Up to 6 hours

До 1 години Up to 1 hour

До 6-12 годин Up to 6-12 hours

96 / 200
Жінка 29-ти років на 32-му тижні вагітності потрапила у ДПТ. При огляді пошкоджень опорно-рухового апарату та внутрішніх органів не виявлено. В якому положенні слід транспортувати постраждалу до лікарні? A 29-year-old woman in the 32nd week of pregnancy was admitted to the DPT. During the examination, no damage to the musculoskeletal system and internal organs was detected. In what position should she be transported the victim to the hospital?

В положенні на правому боці In position on the right side

В положенні Тренделенбурга In Trendelenburg position

В сидячому положенні In a sitting position

В горизонтальному положенні на лівому боці In a horizontal position on the left side

В положенні на спині In the supine position

97 / 200
Чоловіка 48-ми років було виявлено родичами дома без свідомості. Об’єктивно: виражений ціаноз губ, акроціаноз. Подих гучний, із частотою 28/хв. З рота запах алкоголю. При аускультації: подих ослаблено справа на всьому протязі, більше в нижніх відділах. По обидва боки вислухуються сухі й вологі бронхіальні хрипи. Який найбільш імовірний діагноз? A 48-year-old man was found unconscious at home by relatives. Objectively: severe cyanosis of the lips, acrocyanosis. Breathing is loud, with a frequency of 28/min. From the mouth the smell of alcohol. On auscultation: breathing is weakened on the right throughout, more so in the lower parts. Dry and wet bronchial rales are heard on both sides. What is the most likely diagnosis?

Аспірація шлункового вмісту Aspiration of gastric contents

Спонтанний пневмоторакс Spontaneous pneumothorax

Гемоторакс справа Hemothorax on the right

Негоспітальна пневмонія Community-acquired pneumonia

Тромбоемболія легеневої артерії Thromboembolism of the pulmonary artery

98 / 200
Хворий з відкритим переломом стегнової кістки в середній третині поступив до приймального відділення. Об’єктивно: стан важкий, загострені риси обличчя, рясний піт, ціаноз, похолодання нижніх кінцівок, зниження температури тіла, байдужість, дезорієнтація, сонливість, розширення зіниць, тахікардія 140/хв., АТ- 60/40 мм рт.ст., шоковий індекс - 2.0, Hb- 50 г/л, анурія. Якій втраті ОЦК відповідає дана клінічна картина? A patient with an open fracture of the femur in the middle third was admitted to the reception department. Objectively: the condition is severe, sharpened facial features, profuse sweat, cyanosis, coldness of the lower extremities , a decrease in body temperature, indifference, disorientation, drowsiness, pupil dilation, tachycardia 140/min., BP- 60/40 mm Hg, shock index - 2.0, Hb- 50 g/l, anuria. What loss of BCC corresponds to the data clinical picture?

>50% >50%

30-35% 30-35%

20-25% 20-25%

15% 15%

35-40% 35-40%

99 / 200
У хворого після важкої скелетної травми на другу добу раптово виникло зниження артеріального тиску, порушення свідомості та самостійного дихання, дрібноточкові крововиливи на шкірі. Який найбільш імовірний діагноз? On the second day after a severe skeletal injury, the patient suddenly experienced a decrease in blood pressure, impaired consciousness and spontaneous breathing, and small point hemorrhages on the skin. What is the most likely diagnosis?

Жирова емболія Fat embolism

Септичний шок Septic shock

Синдром тривалого стиснення Prolonged compression syndrome

Травматичний шок Traumatic shock

Анафілактичний шок Anaphylactic shock

100 / 200
Дівчинка 6-ти років поступила в стаціонар зі скаргами на підвищення температури до 37,2oC, кровотечу з носа, подразливість, запаморочення. Об’єктивно: шкіра і слизові бліді з іктеричним відтінком, дрібноцяткові висипання на шкірі. На язиці згладженість та атрофія сосочків. Дихання везикулярне. Серцеві тони чисті, ритмічні. Живіт чутливий в епігастрії. Печінка +1 см, селезінка +0,5 см. У крові: Hb- 35 г/л, КП- 0,75, лейкоцити 3,6 • 109 /л, анізо- і пойкілоцитоз. Виберіть оптимальну тактику лікування : A 6-year-old girl was admitted to the hospital with complaints of an increase in temperature to 37.2oC, nosebleeds, irritability, dizziness. Objectively: skin and mucous membranes pale with an icteric hue, small-spotted rashes on the skin. Smoothness and atrophy of the papillae on the tongue. Vesicular breathing. Heart tones are clear, rhythmic. Abdomen is sensitive in the epigastrium. Liver +1 cm, spleen +0.5 cm. In the blood: Hb- 35 g /l, KP- 0.75, leukocytes 3.6 • 109 /l, aniso- and poikilocytosis. Choose the optimal treatment tactics :

Тромбоцитарна маса Platelet mass

Ілюкокортикоїди Ilucocorticoids

Препарати заліза Iron preparations

Переливання крові Blood transfusion

Переливання еритроцитарної маси Red blood cell transfusion

101 / 200
В медичну роту механізованої бригади поступили з поля бою 10 легкопоранених, 15 поранених середньої важкості і 20 тяжкопоранених. Якими принципами та вимогами керується медичний персонал, який виконує сортування поранених в приймальносортувальному відділенні медичної роти? 10 lightly wounded, 15 moderately wounded, and 20 seriously wounded entered the medical company of the mechanized brigade from the battlefield. What are the principles and requirements of the medical personnel who triage the wounded in reception and sorting department of the medical company?

За потребою місця надання медичної допомоги пораненим As needed, provide medical assistance to the wounded

Єдиною класифікацією хвороб і травм та єдиними вимогами до методики сортування Unified classification of diseases and injuries and uniform requirements for the sorting method

За потребою якнайшвидшої евакуації поранених до наступного етапу медичної евакуації For the need of the earliest possible evacuation of the wounded to the next stage of medical evacuation

За потребою розподілу та по важкості стану поранених By the need for distribution and by the severity of the condition of the wounded

Сортування поранених за принципом 'Пироговських рядів' Sorting of the wounded according to the principle of 'Pirogov rows'

102 / 200
У дитячій дошкільній установі серед 120 дітей зареєстровано спалах дизентерії Зонне. На 2-3 добу із середнім ступенем важкості та важкими клінічними формами було госпіталізовано 35 дітей та 4 працівника дитячої установи із різних груп. Вкажіть найбільш імовірний тип спалаху: An outbreak of Zonne's dysentery was registered among 120 children in a children's preschool institution . 35 children and 4 employees were hospitalized for 2-3 days with moderate severity and severe clinical forms of a children's institution from different groups. Indicate the most likely type of outbreak:

Харчовий Food

Контактно-побутовий Contact-domestic

Фекально-оральний Fecal-oral

Водний Water

Контактний Contact

103 / 200
Солдату був в осередку ураження ОР Евакуйований в МПБ, введено підшкірно 4 мл 0,1% розчину атропіну. Скарги на загальну слабкість, спрагу, сухість у горлі, охриплість голосу. Збуджений, поведінка агресивна. Шкіра обличчя гіперемована, зіниці розширені. В легенях жорстке дихання. Ps- И2/хв., АТ-140/90 мм рт.ст. Що могло бути причиною описаної картини? The soldier was in the focus of the OR. He was evacuated to the medical center, 4 ml of 0.1% atropine solution was injected subcutaneously. Complaints of general weakness, thirst, dry throat, hoarseness voice. Excited, aggressive behavior. The skin of the face is hyperemic, the pupils are dilated. Breathing is hard in the lungs. Ps- I2/min., BP-140/90 mm Hg. What could be the cause of the described picture?

Передозування реактиваторів холінесте-рази Overdose of cholinesterase reactivators

Ураження зоманом Infected by soman

Атропінізація організму Atropinization of the body

Ураження зарином Sarin exposure

Ураження дифосгеном Diphosgene exposure

104 / 200
При променевому ураженні шкіри у хворого 33-х років спостерігається вологий дерматит, пухирі малі, ненапружені, займають менш ніж 50% поля. Така клінічна картина виникає при дозі опромінення: In case of radiation damage to the skin, a 33-year-old patient has moist dermatitis, the blisters are small, non-tense, occupying less than 50% of the field. This clinical picture occurs with a dose of radiation :

20-30 Гр 20-30 Gy

5-8 Гр 5-8 Gy

15-20 Гр 15-20 Gy

7-13 Гр 7-13 Gy

10-15 Гр 10-15 Gy

105 / 200
Вагітна 31-го року, 40 тижнів, з бурхливою пологовою діяльністю, клінічно вузьким тазом. Під час індукції наркозу виникло блювання. Після інтубації проведена санація дихальних шляхів та ротової порожнини. В подальшому розвинувся ціаноз, підвищився центральний венозний тиск, над легенями різнокаліберні вологі хрипи, підвищення тиску на вдиху. При контролі газів крові - значна гіпоксемія. Яка причина цього стану? Pregnant in the 31st year, 40 weeks, with violent labor, a clinically narrow pelvis. Vomiting occurred during the induction of anesthesia. After intubation, sanitation of the respiratory tract and oral cavity was carried out cavity. Subsequently, cyanosis developed, central venous pressure increased, moist rales of various calibers over the lungs, increased inspiratory pressure. During blood gas control, significant hypoxemia. What is the cause of this condition?

Емболія навколоплідними водами Amniotic fluid embolism

Синдром Мендельсону Mendelssohn syndrome

Інфаркт міокарда Myocardial infarction

Ендотоксичний шок Endotoxic shock

Набряк легенів Pulmonary edema

106 / 200
Жінка 22 роки, 30 тижнів вагітності, доставлена в інфекційну лікарню з діагнозом гострий гепатит В зі скаргами на різку слабкість, повторне блювання, виражену жовтяницю. При перкусії над печінкою тимпаніт. Яке дослідження треба терміново виконати разом з печінковими пробами? A 22-year-old woman, 30 weeks pregnant, was brought to an infectious disease hospital with a diagnosis of acute hepatitis B with complaints of sharp weakness, repeated vomiting, pronounced jaundice. When percussion over the liver tympanitis. What research should be urgently performed along with liver tests?

HBeAg, aHBe HBeAg, aHBe

Протромбіновий індекс Prothrombin index

Посів блювотних мас Sowing vomit

HBsAg, aHBc HBsAg, aHBc

ПЛР HBV HBV PCR

107 / 200
У стаціонар бригадою швидкої допомоги доставлений потерпілий 49-ти років із ознаками зупинки кровообігу і дихання внаслідок ураження електричним струмом. Після неуспішних реанімаційних заходів, у результаті огляду потерпілого, була ідентифікована локалізація відповідних меток току, викликаних вольтовой дугою, а також світлове ураження очей. На ЕКГ- ізолінія. Які з нижче перерахованих причин привели до розвитку термінального стану? A 49-year-old victim was brought to the hospital by the ambulance crew with signs of circulatory and respiratory arrest due to electric shock. After unsuccessful resuscitation measures, as a result of the victim's examination, it was identified the localization of the corresponding current marks caused by the voltaic arc, as well as light damage to the eyes. An isoline on the ECG. Which of the following causes led to the development of the terminal state?

Тетанічний спазм дихальної мускулатури і голосових зв’язок Tetanic spasm of respiratory muscles and vocal cords

Набряк головного мозку Cerebral edema

Фібриляція шлуночків серця Ventricular fibrillation

Асфіксія Asphyxia

Асистолія Asystole

108 / 200
Хлопчик 7-ми років скаржиться на сильний біль у правій руці. Відомо, що 10 днів тому він впав, під час чого забив праве плече. При огляді пасивні, активні рухи в правій руці обмежені, болючі, набряк, гіперемія, гаряча на дотик шкіра плеча, температура тіла 39,2oC. Який попередній діагноз? A 7-year-old boy complains of severe pain in his right arm. It is known that 10 days ago he fell and bruised his right shoulder. On examination, passive, active movements in the right hand are limited, painful, swelling, hyperemia, the skin of the shoulder is hot to the touch, body temperature is 39.2oC. What is the previous diagnosis?

Флегмона плеча Phlegmon of the shoulder

Остеомієліт плечової кістки Osteomyelitis of humerus

Артрит плечового суглоба Shoulder joint arthritis

Інфікована гематома плеча Infected shoulder hematoma

Забій плеча Shoulder contusion

109 / 200
При проведенні бойових навчань до ПМП був доставлений військовослужбовець 20-ти років. Відомо, що він був знайдений під уламками макету танка. Нижня кінцівка набрякла, шкірні покриви бліді, з великою кількістю подряпин та крововиливів. Підшкірно-жирова клітковина та м’язи ціанотичні, цілісність магістральних судин не порушена. В легенях застійні явища. Який попередній діагноз? During combat training, a 20-year-old serviceman was brought to the PMP. It is known that he was found under the wreckage of a tank model. The lower limb is swollen, the skin is pale, with a large number of scratches and hemorrhages. Subcutaneous adipose tissue and muscles are cyanotic, the integrity of the main vessels is not disturbed. There is congestion in the lungs. What is the previous diagnosis?

Гострий тромбофлебіт Acute thrombophlebitis

- -

Синдром тривалого стиснення Prolonged compression syndrome

Тромбоз нижньої порожнистої вени Thrombosis of the inferior vena cava

Політравма Polytrauma

110 / 200
У хворого 37-ми років, що вживав напередодні копченого ляща, поступово розвинулись м’язова слабкість, неможливість читати газетний текст, на другий день - порушення ковтання. Об’єктивно: стан тяжкий, ціаноз шкіри, страбізм, ЧД- 50/хв., Ps-120/хв. Голос гугнявий, парез зіниць, метеоризм 2 ступеня. Випорожнення відсутні 2 доби. Який лікувальний захід показаний у першу чергу? A 37-year-old patient, who had consumed smoked bream the day before, gradually developed muscle weakness, the inability to read a newspaper text, and on the second day - swallowing disorders. About objectively: the condition is severe, cyanosis of the skin, strabismus, BH-50/min., Ps-120/min. Voice hoarse, paresis of the pupils, flatulence of the 2nd degree. There are no bowel movements for 2 days. What medical measure is indicated first of all?

Штучне дихання 'рот у рот' Mouth-to-mouth artificial respiration

Введення діуретиків Introduction of diuretics

Призначення прозерину Destination of proserin

Трахеостомія Tracheostomy

Штучна вентиляція легень Artificial lung ventilation

111 / 200
Після проходження радіаційної хмари, у приймальне відділення був привезений хлопчик 10- ти років зі скаргами на нудоту, дворазове блювання, головний біль. При огляді: шкіра бліда, ЧСС- 105/хв., АТ- 120/80 мм рт.ст., свідомість не втрачена. Показники первинної реакції при зовнішньому рівномірному опроміненні - 3 Гр. Визначте обсяг надання первинної допомоги: After the passage of the radiation cloud, a 10-year-old boy was brought to the reception department with complaints of nausea, vomiting twice, headache. On examination: pale skin, heart rate - 105/min., blood pressure - 120/80 mm Hg, consciousness is not lost. Indicators of the primary reaction to external uniform irradiation - 3 Gy. Determine the scope of providing primary care:

Введення протиблювотних засобів та госпіталізація Introduction of antiemetics and hospitalization

Ізоляція Isolation

Введення сорбентів і госпіталізація Introduction of sorbents and hospitalization

Госпіталізація до інфекційного відділення Hospitalization to the infectious department

Введення протиблювотних засобів та аналгетиків Introduction of antiemetics and analgesics

112 / 200
В гематологічному відділенні лікарні знаходиться хворий 41-го року, який скаржиться на відчуття тривоги, страху, погане загальне самопочуття, біль у животі та грудній клітці, відчуття серцебиття, задишку. Об’єктивно: гіперемія шкіри, температура тіла 38,4oC, АТ90 /60 мм рт.ст, тахікардія, у легенях вологі хрипи, живіт під час пальпації чутливий, особливо в ділянці правого підребер’я. Відомо, що 3 години тому, хворому було проведене переливання еритро-цитарної маси. Введення яких препаратів є патогенетично обґрунтованим в даному випадку? There is a 41-year-old patient in the hematology department of the hospital who complains of anxiety, fear, poor general well-being, pain in the abdomen and chest, palpitations, shortness of breath. Objectively: hyperemia of the skin, body temperature 38.4oC, blood pressure 90/60 mmHg, tachycardia, wet rales in the lungs, the abdomen is sensitive during palpation, especially in the area of ​​the right hypochondrium. It is known that 3 hours ago , the patient underwent transfusion of erythrocyte mass. The administration of which drugs is pathogenetically justified in this case?

Антигістамінні Antihistamines

Глюкортикоїди Glucorticoids

Інгібітори ЦОГ-2 COX-2 inhibitors

Нестероїдні протизапальні препарати Nonsteroidal anti-inflammatory drugs

Антигіпертензивні препарати Antihypertensive drugs

113 / 200
Хлопчик 12-ти років після ДТП. Скарги на запаморочення. Об’єктивно: на промежині гематома, з уретри виділяються краплі крові. Який метод діагностики перелому кісток тазу є першочерговим на фоні протишокових заходів? A 12-year-old boy after a road accident. Complains of dizziness. Objectively: hematoma on the perineum, drops of blood are coming out of the urethra. What is the method of diagnosing a fracture of the pelvic bones priority against the background of anti-shock measures?

Рентгенографія органів тазу X-ray of pelvic organs

Висхідна уретрографія Ascending urethrography

Ректальне пальцеве обстеження Rectal digital examination

Ультразвукове обстеження Ultrasound examination

Цистоскопія Cystoscopy

114 / 200
Хворий 18-ти років, в анамнезі черепно-мозкова травма, раптово впав, знепритомнів, тоніко-клонічні судоми, кровава піна з рота. Обличчя бліде з ціанотичним відтінком, зіниці на світло не реагують. Такий стан біля 3 хвилин. Яка необхідна невідкладна допомога? An 18-year-old patient with a history of craniocerebral trauma, suddenly fell, fainted, tonic-clonic convulsions, bloody foam from the mouth. The face is pale with a cyanotic tint , the pupils do not react to light. This condition lasts for about 3 minutes. What is the need for emergency help?

Введення хлоралгідрату per rectum Introduction of chloral hydrate per rectum

Одноразове введення седуксену One-time injection of seduxen

Введення 40% розчину глюкози Introduction of 40% glucose solution

Введення тиопенталу натрію Introduction of sodium thiopental

Оксигенотерапія Oxygenotherapy

115 / 200
Чоловік 39-ти років під час бійки отримав удар по шиї ребром долоні. Внаслідок травми знепритомнів, дихання різко утруднене, із утрудненим вдихом, шкіра обличчя та рук сіросинього кольору. Що треба терміново зробити хворому? During a fight, a 39-year-old man was hit on the neck with the edge of a palm. As a result of the injury, he fainted, breathing was difficult, with difficulty breathing, the skin of his face and hands was gray-blue What should be done urgently to the patient?

Інтубація трахеї Tracheal intubation

Інтубація, трахеостомія Intubation, tracheostomy

Трахеотомія Tracheotomy

Конікотомія Conicotomy

Трахеостомія Tracheostomy

116 / 200
Хворій 30-ти років проводиться непрямий масаж серця та ШВЛ у продовж 2-3 хвилин, але пульсація на a.carotis не визначається. Який наступний крок треба зробити в умовах операційної? A 30-year-old patient undergoes indirect heart massage and mechanical ventilation for 2-3 minutes, but the pulsation on the a. carotid is not determined. What next step should be taken in operating room conditions?

Адреналін внутрішньосерцево Adrenaline intracardiac

Прямий масаж серця Direct heart massage

Продовжувати непрямий масаж серця Continue indirect heart massage

Припинити серцево-легеневу реанімацію Stop CPR

Адреналін в/в Adrenaline IV

117 / 200
Хворій 26-ти років з гострим лейкозом було перелито 200 мл іншогрупної еритро-цитарної маси. У хворої з’явилися інтенсивні болі у попереку, катетером з сечового міхура виведено 100 мл сечі червоно-лакового кольору. Яке ускладнення розвивається у хворої? A 26-year-old patient with acute leukemia was transfused with 200 ml of non-group erythrocyte mass. The patient developed intense pain in the lower back, a catheter was removed from the bladder 100 ml of red-lacquered urine. What complication is developing in the patient?

Гостра ниркова недостатність Acute renal failure

Радикуліт Sciatica

Інфаркт нирки Kidney infarction

Гострий гломерулонефрит Acute glomerulonephritis

Ниркова колька Renal colic

118 / 200
У хворої 35-ти років на фоні лихоманки і болю в горлі під час огляду ротоглотки виявлені плівчасті сірі нашарування на мигдаликах, що розповсюджується на піднебінну дужку, набряк, гіперемія і кровоточивість слизової. Назвіть першочерговий препарат для лікування хворої: A 35-year-old patient, against the background of fever and sore throat, during an examination of the oropharynx, membranous gray layers on the tonsils, spreading to the palatal arch, edema, hyperemia were found and bleeding of the mucous membrane. Name the primary drug for the treatment of the patient:

Протидифтерійна сироватка Anti-diphtheria serum

Офлоксацин Ofloxacin

Доксіциклін Doxycycline

Пеніцилін Penicillin

Макропен Macropen

119 / 200
Хворий 29-ти років протягом останнього тижня скаржиться на періодичні озноби, підвищення температури до 39oC, рясну пітливість. При огляді: температура 36,2oC, блідий, склери жовтяничні, спленомегалія. Два тижні потому повернувся з !ндії. Який попередній діагноз? A 29-year-old patient has been complaining of periodic chills for the past week, an increase in temperature up to 39oC, profuse sweating. On examination: temperature 36.2oC, pale, sclera are jaundiced , splenomegaly. He returned from India two weeks later. What was the previous diagnosis?

Малярія Malaria

Вірусний гепатит Е Viral hepatitis E

Жовта лихоманка Yellow fever

Лептоспіроз Leptospirosis

Лейшманіоз Leishmaniasis

120 / 200
В результаті ДТП у 6-річного хлопчика має місце важкий стан (пошкодження грудей, живота та кінцівок), шкіра з 'марму-ровим'малюнком, акроціаноз, тахікардія, пульс слабкого наповнення і напруження, АТ низький. Здійснюються невідкладні заходи після відновлення ОЦК. Дитині показано введення: As a result of a road accident, a 6-year-old boy has a serious condition (injuries to the chest, abdomen and limbs), skin with a 'marble moat' pattern, acrocyanosis, tachycardia , pulse of weak filling and tension, blood pressure is low. Urgent measures are taken after recovery of BCC. The child is shown the introduction:

Дофаміну Dopamine

Седативних препаратів Sedative drugs

Гангліоблокаторів Ganglioblockers

Препаратів калію Potassium drugs

Серцевих глікозидів Cardiac glycosides

121 / 200
Хвора 33-х років захворіла гостро, коли з’явились озноб, нудота, підвищилась температура тіла до 39,3oC, наростала слабкість. На 2 день хвороби з’явився висип на шкірі. Об’єктивно: гіперемія обличчя, дрі-бнокрапчастий геморагічний висип в над, підключичній та пахвовій ділянках, різко позитивний симптом Пастернацького з обох боків. В сечі: білок, циліндри, еритроцити. Яке захворювання є найбільш імовірним? A 33-year-old patient became acutely ill, when chills, nausea appeared, the body temperature rose to 39.3oC, and weakness increased. On the 2nd day of the illness with a rash appeared on the skin. Objectively: hyperemia of the face, small-spotted hemorrhagic rash in the supraclavicular, subclavian and axillary areas, sharply positive Pasternacki's symptom on both sides. In the urine: protein, cylinders, erythrocytes. What disease is most likely?

Сепсис Sepsis

Висипний тиф Typhoid

Вітряна віспа Chicken Pox

Менінгококова інфекція Meningococcal infection

Геморагічна гарячка з нирковим синдромом Hemorrhagic fever with renal syndrome

122 / 200
Чоловік 30-ти років звільнений з ув’язнення. За день до звільнення був побитий чотирма співкамерниками та зґвалтований. Похмурий, тривожний, пригнічений. В сновидіннях і спогадах яскраво, до дрібниць відтворюються епізоди перенесеного насильства. Все, що нагадує про пережиту травму, викликає ці спогади з вибухом дратівливості. Відчуття самотності. Розвиток якого психічного розладу слід очікувати? A 30-year-old man has been released from prison. The day before his release, he was beaten by four inmates and raped. Gloomy, anxious, depressed. In dreams and memories vividly , the episodes of the experienced violence are recreated in detail. Anything that reminds of the trauma experienced causes these memories with an explosion of irritability. A feeling of loneliness. The development of which mental disorder should be expected?

Рекурентний депресивний розлад Recurrent depressive disorder

Генералізований тривожний розлад Generalized Anxiety Disorder

Посттравматичний стресовий розлад Post-traumatic stress disorder

Невротичний розвиток особистості Neurotic personality development

Пролонгована депресивна реакція Prolonged depressive reaction

123 / 200
Жінка 38-ми років з 14-ти років страждає на генералізовані судомні напади. Звернулася до дільничого терапевта з приводу підвищення температури, нежиті й кашлю. Під час огляду стає напруженою, перестає відповідати на запитання, з’являється вираз жаху на обличчі, раптово кидається бігти. Яка тактика організації надання допомоги? A 38-year-old woman has been suffering from generalized convulsive attacks since the age of 14. She consulted a district therapist because of a fever, runny nose, and cough. During the examination, she becomes tensed, stops answering questions, has a look of horror on her face, suddenly starts running. What are the tactics of the aid organization?

Введення транквілізаторів, госпіталізація в психіатричну лікарню Introduction of tranquilizers, hospitalization in a psychiatric hospital

Введення транквілізаторів, доставка до дільничого психіатра Introduction of tranquilizers, delivery to the district psychiatrist

Призначення антиконвульсантів, спостереження в динаміці Prescription of anticonvulsants, observation in dynamics

Введення нейролептиків, госпіталізація в неврологічне відділення Introduction of neuroleptics, hospitalization in the neurological department

Трансфузія реосорбілакту, госпіталізація в реанімаційне відділення Transfusion of reosorbilact, hospitalization in the intensive care unit

124 / 200
Хвору 24-х років турбують переймоподібні болі у нижніх відділах живота, які передують позивам до дефекації. Клінічно -випорожнення з домішками слизу, гною та крові. Хворіє 10 років, останнє загострення впродовж тижня. Випорожнення 8-10 раз на добу. Об’єктивно: температура тіла 37, 7oC, живіт під час пальпації м’який, болючий у лівій клубовій ділянці. Який найбільш імовірний діагноз? A 24-year-old patient suffers from spasm-like pains in the lower abdomen, which precede the urge to defecate. Clinically, defecation with impurities of mucus, pus and blood. He has been ill for 10 years , the last exacerbation within a week. Defecation 8-10 times a day. Objectively: body temperature 37.7oC, the abdomen during palpation is soft, painful in the left iliac region. What is the most likely diagnosis?

Неспецифічний виразковий коліт Nonspecific ulcerative colitis

Хвороба Крона Crohn's disease

Холера Cholera

Синдром подразненої кишки Irritable bowel syndrome

Пухлина товстої кишки Colon tumor

125 / 200
Чоловікові близько 65-ти років. Стрибав у воду з причалу. Стан після втоплен-ня. На березі йому розпочинають серцево-легеневу реанімацію. Які особливості техніки реанімації у даному випадку? The man is about 65 years old. He jumped into the water from the wharf. His condition after drowning. On the shore, cardiopulmonary resuscitation is started on him. What are the features of resuscitation techniques in in this case?

Не виконують закидання голови Don't perform head tossing

Завжди виконують прийом Геймліха Always perform the Heimlich maneuver

Штучна вентиляція легень проводиться подвійним дихальним об’ємом Artificial lung ventilation is performed with a double respiratory volume

Не виконують виведення нижньої щелепи The mandible is not removed

Непрямий масаж серця розпочинають раніше, ніж штучну вентиляцію легень Indirect heart massage is started earlier than artificial lung ventilation

126 / 200
Хворий 26-ти років знаходиться у відділенні з приводу гострого вірусного гепатиту В, стан погіршився на 20-й день хвороби: з’явилися біль у животі, нудота, наросла жовтяниця. Вночі не спав, збуджений, нечітко орієнтується у часі. Печінка пальпується біля краю реберної дуги, м’якої консистенції. Що зумовлює стан хворого? A 26-year-old patient is in the department for acute viral hepatitis B, his condition worsened on the 20th day of the illness: abdominal pain, nausea, jaundice has developed. He did not sleep at night, he is excited, he is not clearly oriented in time. The liver is palpable near the edge of the costal arch, soft in consistency. What causes the patient's condition?

Розвиток інфекційно-токсичного шоку Development of infectious-toxic shock

Розвиток печінкової енцефалопатії Development of hepatic encephalopathy

Розвиток гострого панкреатиту Development of acute pancreatitis

Є характерним для перебігу гострого вірусного гепатиту В It is typical for the course of acute viral hepatitis B

Прийом наркотичних речовин Taking drugs

127 / 200
У хворого 80-ти років під час оперативного втручання з приводу защемленої кили наступила асистолія. Розпочато непрямий масаж серця, ШВЛ. Які ускладнення найчастіше трапляються під час проведення серцево-легеневої реанімації у таких хворих, що не є перепоною для відновлення життєвих функцій? An 80-year-old patient experienced asystole during surgery for a pinched hernia. Indirect heart massage, mechanical ventilation was started. What complications most often occur during cardiopulmonary resuscitation pulmonary resuscitation in such patients, which is not an obstacle to the restoration of vital functions?

Крововиливи та некроз міокарда Hemorrhages and necrosis of myocardium

Розрив селезінки Rupture of the spleen

Перелом ребер чи відрив хрящів Rib fracture or cartilage tear

Розрив печінки Liver rupture

Розрив шлунково-кишкового тракту Rupture of the gastrointestinal tract

128 / 200
У матері та хлопчика 7-ми років гостро розвинулися часті водянисті випорожнення за типом 'рисового відвару', біль у животі відсутній. Температура тіла дитини 35, 5oC, виражені ознаки зневоднення. З метою профілактики захворювання в контактних членів сім’ї слід: A mother and a 7-year-old boy acutely developed frequent watery stools of the 'rice broth' type, there is no abdominal pain. The child's body temperature is 35.5oC, pronounced signs of dehydration. In order to prevent the disease, contact family members should:

Провести опромінення приміщень бактерицидними лампами Irradiate the premises with bactericidal lamps

Запровадити вдома заходи по боротьбі із мухами Introduce fly control measures at home

Ізолювати їх та призначити антибіотики Isolate them and prescribe antibiotics

Рекомендувати кип’ятіння питної води Recommend boiling drinking water

Запровадити масковий режим Enable mask mode

129 / 200
Хворий 24-х років скаржиться на головний біль, нудоту, блювання, зниження гостроти зору. Скарги з’явилися після перенесеної рік тому черепно-мозкової травми. Об’єктивно: напівптоз, легка асиметрія ви-шкіру зубів зліва. Глибокі рефлекси симетрично підвищені з обох сторін. М’язовий тонус незначно підвищений за пластичним типом. Яке інструментальне обстеження необхідно провести з метою уточнення діагнозу? A 24-year-old patient complains of headache, nausea, vomiting, decreased visual acuity. Complaints appeared after a craniocerebral injury suffered a year ago. About' Objectively: semiptosis, slight asymmetry of the upper and lower teeth on the left. Deep reflexes are symmetrically increased on both sides. Muscle tone is slightly increased according to the plastic type. What instrumental examination should be performed in order to clarify the diagnosis?

Електроміографія Electromyography

Електроенцефалографія Electroencephalography

Викликані потенціали Evoked potentials

Ехо-енцефалоскопія Echo-encephaloscopy

Магнітно-резонансна томографія Magnetic resonance imaging

130 / 200
Хворий 35-ти років протягом 10-ти років зловживає алкоголем. П’ять днів тому випив невідому спиртову речовину. Об’єктивно: дезорієнтований, рухи безладні, аморфні, хаотичні в межах ліжка. Викрикує окремі слова, склади, звуки. Соматичний стан вкрай важкий. Язик сухий, температура тіла 40 — 41oC, на тілі петехії. Невідкладна допомога повинна надаватися в: A 35-year-old patient has been abusing alcohol for 10 years. Five days ago he drank an unknown alcoholic substance. Objectively: disoriented, movements are disordered, amorphous , chaotic within the bed. Exclaims individual words, syllables, sounds. The somatic condition is extremely difficult. The tongue is dry, the body temperature is 40 — 41oC, petechiae on the body. Emergency care should be provided in:

Наркологічній лікарні Narcological hospital

Неврологічному стаціонарі To a neurological hospital

Соматичному стаціонарі To a somatic hospital

Психіатричному відділенні Psychiatric Department

Реанімаційному відділенні Intensive department

131 / 200
Хвора 59-ти років у ступорозному стані. При об’єктивному обстеженні: температура тіла 38,3oC, шкірні покриви жовтяничні, наявні підшкірні крововиливи. ЧД- 27/хв., Ps- 116/хв., АТ- 80/50 мм рт.ст. Печінка виступає з-під краю правої реберної дуги на 4 см. Що з даних лабораторного дослідження буде свідчити про наростаючу гостру печінкову недостатність? A 59-year-old patient is in a stupor. On objective examination: body temperature is 38.3oC, the skin is jaundiced, there are subcutaneous hemorrhages. ChD- 27/ min., Ps- 116/min., BP- 80/50 mm Hg. The liver protrudes from under the edge of the right costal arch by 4 cm. Which of the laboratory test data will indicate an increasing acute liver failure?

Гіперхолестеринемія Hypercholesterolemia

Підвищення рівня фібриногену Increased fibrinogen level

Зниження протромбінового індексу Decreased prothrombin index

Підвищення протромбінового індексу Prothrombin index increase

Гіперальбумінемія Hyperalbuminemia

132 / 200
Хвора 57-ми років 18 років страждає на варикозне розширення вен нижніх кінцівок, неодноразово хворіла на тромбофлебіт. Від оперативного лікування відмовляється. При огляді на обох стегнах звиті потовщені вени. По ходу однієї з них болісність та ущільнення із запаленням шкіри. Шкіра нижньої третина стегон темнокори-чневого кольору, щільна, у складку не береться. Розвиток якого ускладнення загрожує хворій? A 57-year-old patient has been suffering from varicose veins of the lower extremities for 18 years, has repeatedly suffered from thrombophlebitis. She refuses surgical treatment. On examination, thickened veins are found on both thighs In the course of one of them, painfulness and tightness with inflammation of the skin. The skin of the lower third of the thighs is dark brown, dense, does not fold. What complication is the patient at risk of developing?

Розвиток гангрени кінцівок Development of limb gangrene

Утворення трофічних виразок Formation of trophic ulcers

Перехід запалення на глибокі вени Transition of inflammation to deep veins

Виникнення тромбозу глибоких вен Occurrence of deep vein thrombosis

Синя флегмазія Blue Phlegmasia

133 / 200
Хворий 56-ти років доставлений в тяжкому стані. Об’єктивно: свідомість сплутана. Іктеричність склер, 'судинні зірочки'на шкірі кінцівок, гіперемія долоней. Із рота 'печінковий'запах. АТ- 90/50 мм рт.ст. Рs-100/хв. Живіт збільшений, помітні розширені вени, пупок видається. Перкуторно визначається асцит. Печінка збільшена на 6 см. Яка причина погіршення стану хворого? A 56-year-old patient was brought in in serious condition. Objectively: consciousness is confused. Icteric sclera, 'vascular stars' on the skin of the extremities, hyperemia of the palms. From the mouth 'hepatic' smell. Blood pressure - 90/50 mm Hg. Rs-100/min. Abdomen is enlarged, dilated veins are visible, the navel is visible. Ascites is determined by percussion. The liver is enlarged by 6 cm. What is the reason for the deterioration of the patient's condition?

Постгіпоксична енцефалопатія Post hypoxic encephalopathy

Гостра печінкова недостатність Acute liver failure

Метаболічний алкалоз Metabolic alkalosis

Гіпоглікемічна кома Hypoglycemic coma

Гостра ниркова недостатність Acute renal failure

134 / 200
У 15-річного підлітка, хворого на артеріальну гіпертензію, раптово виникли біль голови, запаморочення, дзвін у вухах, нудота, блювання. АТ- 180/100 мм рт.ст. Діагностовано гіпертензивний криз. Який препарат слід призначити в першу чергу? A 15-year-old teenager suffering from arterial hypertension suddenly developed a headache, dizziness, ringing in the ears, nausea, vomiting. BP- 180/100 mm Hg Art. A hypertensive crisis has been diagnosed. What drug should be prescribed first?

Кордарон Cordaron

Ніфедипін під язик Sublingual Nifedipine

Еналаприл Enalapril

Нормотенс Normotens

Гіпотіазид Hypotiazid

135 / 200
Хворого 27-ми років доставлено у від- ділення реанімації в критичному стані. Констатовано геморагічний шок. Який захід найбільш ефективний для забезпечення адекватної інфузійної терапії? A 27-year-old patient was brought to the intensive care unit in critical condition. Hemorrhagic shock was diagnosed. What is the most effective measure to ensure adequate infusion therapy?

Катетеризація двох кубітальних вен Catheterization of two cubital veins

Пункція кубітальної вени Cubital vein puncture

Катетеризація центральної вени Central venous catheterization

Катетеризація кубітальної вени Cubital vein catheterization

Венесекція Venesection

136 / 200
Хворому 22-х років з політравмою в реанімаційній залі проводиться серцево-легеневомозкова реанімація. Який критерій ефективності закритого масажу серця має найбільшу прогностичну цінність? A 22-year-old patient with polytrauma is undergoing cardiopulmonary resuscitation in the intensive care unit. Which criterion for the effectiveness of closed heart massage has the greatest prognostic value?

Аускультативно вислуховуються тони серця Heart tones are auscultated

Відновлення самостійного дихання Restoration of independent breathing

Наявність пульсації на сонній артерії Presence of pulsation on the carotid artery

Наявність пульсу на променевої артерії Presence of a pulse on the radial artery

Звуження зіниць Pupillary Constriction

137 / 200
Сімейний лікар оглянув хвору 39-ти років зі скаргами на напад гострого болю за грудниною, серцебиття, запаморочення, нудоту, слабкість. На ЕКГ: ЧСС- 230/хв., зубець P відсутній, шлуночкові комплекси деформовані. Який препарат доцільно призначити у якості першочергового на дого-спітальному етапі? The family doctor examined a 39-year-old patient with complaints of an attack of sharp pain behind the sternum, palpitations, dizziness, nausea, weakness. On the ECG: heart rate - 230/min ., the P wave is absent, the ventricular complexes are deformed. What drug should be prescribed as the first priority at the pre-hospital stage?

Ізоптин Isoptin

Новокаїнамід Novocaineamide

Лідокаїн Lidocaine

Морфін Morphine

Строфантин Strophantin

138 / 200
Хлопчик 5-ти років доставлений до приймального відділення після вживання оцтової есенції в невідомій кількості. Об’єктивно: загальний стан середньої важкості, збуджений, АТ- 120/80 мм рт.ст., Ps- 97/хв. Після реанімаційних заходів дитині необхідно: A 5-year-old boy was brought to the reception department after consuming an unknown amount of vinegar essence. Objectively: general condition of medium severity, excited, blood pressure 120/80 mm Hg, Ps- 97/min. After resuscitation measures, the child needs:

Виконати рентгенографію органів грудної клітки Perform chest X-ray

Виконати ФЕГДС Execute FEGDS

Призначити нагляд та ліжковий режим Assign supervision and bed rest

Нейтралізувати шлунковий вміст лугом Neutralize stomach contents with alkali

- -

139 / 200
У чоловіка 29-ти років після роботи в зоні витоку азотної кислоти розвинувся токсичний набряк легень. Яка першочергова допомога? A 29-year-old man developed toxic pulmonary edema after working in a nitric acid leak area. What is the first aid?

Штучна вентиляція легень Artificial lung ventilation

В/в 400 мл 5% розчину глюкози In/in 400 ml of 5% glucose solution

Оксигенотерапія з інгаляцією етилового спирту Oxygen therapy with inhalation of ethyl alcohol

В/в 400 мл реосорбілакту IV 400 ml of reosorbilact

В/в 400 мл фізіологічного розчину In/in 400 ml of physiological solution

140 / 200
Який об’єм хірургічної допомоги слід провести пораненому з крапковими сліпими і наскрізними проникаючими пораненнями легень на етапі кваліфікованої медичної допомоги, коли відсутні симптоми вираженої внутрішньої кровотечі та пневмотораксу, що наростає? What volume of surgical care should be provided to a wounded person with punctate, blind and penetrating lung wounds at the stage of qualified medical care, when there are no symptoms of pronounced internal bleeding and pneumothorax, which is increasing?

Проводиться ушивання ранових отворів Wound holes are being sutured

Слід провести екстрену торакотомію з боку ушкодження Emergency thoracotomy should be performed on the side of the injury

Виконують плевральні пункції з введенням антибіотиків широкого спектру дії They perform pleural punctures with the introduction of broad-spectrum antibiotics

Ці поранені хірургічного втручання не потребують These wounded do not need surgery

Проводиться дренування плевральної порожнини The pleural cavity is being drained

141 / 200
На заводі розірвалась ємність з білим фосфором, в результаті чого 10 працівників одержали опіки. Яку допомогу необхідно надати постраждалим негайно? At the plant, a container with white phosphorus burst, as a result of which 10 workers received burns. What help should be provided to the victims immediately?

Промивання опіків фізрозчином Washing burns with saline

Промивання опіків кислотою Washing burns with acid

Промивання опіків розчином мідного купоросу Washing of burns with a solution of copper sulphate

Промивання опіків водою Washing burns with water

Промивання опіків спиртом Washing burns with alcohol

142 / 200
В якому положенні хворого доцільно проводити заходи серцево-легеневої реанімації? In what position of the patient is it advisable to perform cardiopulmonary resuscitation?

В положенні на боці Side position

В положенні Тренделенбурга In Trendelenburg position

На спині в горизонтальному положенні On the back in a horizontal position

Положення хворого не має суттєвого значення The position of the patient is not important

На спині з піднесеним головним кінцем On the back with the head end raised

143 / 200
Дитина 7-ми років скаржиться на лихоманку, пронос, блювання та запаморочення. У крові: Ht- 0,55, Na+-118 ммоль/л, К+- 5,9 ммоль/л. Які розлади водно-електролітного балансу спостерігаються? A 7-year-old child complains of fever, diarrhea, vomiting and dizziness. In the blood: Ht- 0.55, Na+-118 mmol/l, K+ - 5.9 mmol/l. What disorders of the water-electrolyte balance are observed?

Гіпотонічна дегідратація, гіпокаліємія Hypotonic dehydration, hypokalemia

Гіпотонічна дегідратація, гіперкаліємія Hypotonic dehydration, hyperkalemia

Гіперкаліємія Hyperkalemia

Ізотонічна дегідратація, гіперкаліємія Isotonic dehydration, hyperkalemia

Розладів водно-електролітного балансу немає There are no water-electrolyte balance disorders

144 / 200
У хлопчика 10-ти років на 5-й день лікування гепарином з приводу шкірно-суглобової форми хвороби Шенляйн-Іеноха, посилилась геморагічна висипка, з’явились м’язові гематоми, гемоптое, мелена. Активований парціальний тромбопла-стиновий час - 4 хвилини. Який препарат доцільно призначити хлопчику? In a 10-year-old boy, on the 5th day of heparin treatment for the cutaneous-articular form of Schoenlein-Ienoch's disease, the hemorrhagic rash increased, m ulcerative hematomas, hemoptysis, melena. Activated partial thromboplastin time - 4 minutes. What drug should be prescribed to the boy?

Строфантин Strophantin

Протаміну сульфат Protamine sulfate

Активоване вугілля Activated carbon

Натрію гідрокарбонат Sodium bicarbonate

Ілюконат кальцію Calcium Iluconate

145 / 200
Пологи у 42 тижня. Вагітність перебігала на тлі дифузного зобу II А стадії, обтя-женого акушерського анамнезу, ускладнена дистресом плоду в пологах. Передні та задні навколоплідні води містять частки меконію. Новонароджений хлопчик з масою 4000 г, довжиною 56 см. При народженнї вїдсутня функцїя зовнішнього дихання, значно знижений м’язовий тонус, ЧСС- 90/хв. Вкажіть першу дію у послідовності заходів первинної реанімації новонародженого у даному випадку: Delivery at the 42nd week. The pregnancy took place against the background of diffuse goiter II A stage, heavy obstetric history, complicated by fetal distress during childbirth. The anterior and posterior amniotic fluids contain particles of meconium. A newborn boy with a mass of 4000 g, length of 56 cm. At birth, the function of external breathing is absent, muscle tone is significantly reduced, heart rate - 90/min. Specify the first action in the sequence of measures for primary resuscitation of a newborn in this case:

Корекція гіповолемії Correction of hypovolemia

Штучний масаж серця Artificial heart massage

Санація верхніх дихальних шляхів Upper respiratory tract sanitation

Киснева допомога Oxygen assistance

Тактильна стимуляція Tactile stimulation

146 / 200
Хлопчик 4-х років хворіє на менінгоко-кцемію впродовж останніх 6 годин: температура тіла 40oC, геморагічний висип поширився з обличчя на усі ділянки шкіри. Різко загальмований, ціаноз шкіри. Кінцівки холодні, дихання поверхневе, 20/хв. ЧСС-58/хв., АТ65 /35 мм рт.ст. У крові: лейкоцити - 6 Г/л, ШЗЕ- 5 мм/год. BE = -12 мМ/л. Ліквор - норма. Знепритомнів. Прогноз для дитини: A 4-year-old boy has been suffering from meningococcytemia for the past 6 hours: body temperature 40oC, a hemorrhagic rash has spread from the face to all areas of the skin. He is severely retarded, cyanosis skin. Cold extremities, shallow breathing, 20/min. HR-58/min., AT65 /35 mm Hg. In the blood: leukocytes - 6 G/l, SZE - 5 mm/h. BE = -12 mm/ l. Liquor is the norm. Fainted. Prognosis for the child:

Несприятливий стосовно одужання Unfavorable for recovery

Прогнозована летальність менше 50% Predicted fatality rate is less than 50%

Прогнозована летальність майже 100% Predicted fatality is almost 100%

Сприятливий стосовно життя Favorable about life

Неможливо оцінити за цими даними Unable to estimate based on these data

147 / 200
Хворий 64-х років тривалий час хворіє на ІХС, скаржиться на сильний загрудин-ний біль, що не купується прийомом таблеток нітрогліцерину. Об’єктивно: свідомість сплутана, шкірні покриви бліді, акроціаноз, холодний липкий піт, дихання поверхневе часте, Ps100 /хв, ниткоподібний, АТ- 80/50 мм рт.ст. Як розцінити стан хворого? A 64-year-old patient has been suffering from coronary heart disease for a long time, complains of severe chest pain, which is not relieved by taking nitroglycerin tablets. Objectively: consciousness is confused, the skin is pale, acrocyanosis, cold sticky sweat, shallow breathing is frequent, Ps100/min, filamentous, blood pressure - 80/50 mm Hg. How to assess the patient's condition?

Порушення мозкового кровообігу Disruption of cerebral circulation

Колапс Collapse

Напад пароксизмальної шлуночкової тахікардії Attack of paroxysmal ventricular tachycardia

Кардіогенний шок Cardiogenic shock

Тромбоемболія легеневої артерії Thromboembolism of the pulmonary artery

148 / 200
Хвора 60-ти років перенесла холецисте-ктомію з приводу деструктивного холециститу. Впродовж 15-ти днів у комплексному лікуванні отримувала гентаміцин по 400 мг 3 рази в день. В анамнезі - патології нирок немає, проте у пацієнтки наявні симптоми гострої ниркової недостатності. Добовий діурез 1,5 л, креатинін 0,63 ммоль/л, питома густина сечі 1004. Артеріальний тиск у межах норми. При ультрасонографії черевної порожнини патологічних змін не виявлено. Яка причина ниркової недостатності? A 60-year-old patient underwent cholecystectomy due to destructive cholecystitis. For 15 days, she received gentamicin 400 mg 3 times a day in complex treatment. anamnesis - there is no kidney pathology, but the patient has symptoms of acute renal failure. Daily diuresis 1.5 L, creatinine 0.63 mmol/L, specific gravity of urine 1004. Blood pressure within the normal range. Abdominal ultrasonography revealed no pathological changes. What is the cause of kidney failure?

Гострий гломерулонефрит Acute glomerulonephritis

Інтраопераційне пошкодження уретри Intraoperative damage to the urethra

Правобічний обструктивний уролітіаз Right-sided obstructive urolithiasis

Нефротоксичність гентаміцину Gentamicin nephrotoxicity

Нефротичний синдром Nephrotic syndrome

149 / 200
Чоловік 30-ти років, різнороб, скаржиться на світлобоязнь, сльозотечу, зниження гостроти зору обох очей. Вчора допомагав у роботі під час електрозварювання, захи- сним щитком не користувався. Гострота зору обох очей 0,9. Об’єктивно: легка світлобоязнь. Рогівка фарбується флюоресцеїном у вигляді окремих крапок. Чутливість рогівки знижена. Інші відділи ока не змінені. Який найбільш імовірний діагноз? A 30-year-old man, a handyman, complains of photophobia, lacrimation, decreased visual acuity in both eyes. Yesterday he helped with the work during electric welding, he did not wear a protective shield used. Visual acuity of both eyes 0.9. Objectively: mild photophobia. The cornea is stained with fluorescein in the form of separate dots. The sensitivity of the cornea is reduced. Other parts of the eye are not changed. What is the most likely diagnosis?

Інфекційний кератит Infectious keratitis

Помутніння склистого тіла Opacity of the vitreous body

Електроофтальмія Electrophthalmia

Помутніння рогівки Opacity of the cornea

Вікова катаракта Age cataract

150 / 200
Хворий 51-го року доставлений у відділення кардіореанімації (інтенсивної терапії), скаржиться на тривалий пекучий за-грудинний біль, який виник 40 хвилин тому, задишку. Об’єктивно: в легенях дихання з жорстким відтінком, тони серця різко ослаблені, АТ-110/70 мм рт.ст. На ЕКГ виявлено підйом сегменту ST на 5 мм над ізолінією у відведеннях I, V1 — V4 та депресію сегмента ST у відведеннях III і aVF. Який препарат потрібно застосувати у даній ситуації? A 51-year-old patient was brought to the cardioresuscitation (intensive care) department, complaining of long-lasting burning pain behind the chest that started 40 minutes ago, shortness of breath. About' objectively: in the lungs, breathing with a hard tone, heart sounds are sharply weakened, BP-110/70 mm Hg. On the ECG, an elevation of the ST segment by 5 mm above the isoline in leads I, V1 — V4 and depression of the ST segment in leads III and aVF. What drug should be used in this situation?

Метопролол Metoprolol

Нітрогліцерин Nitroglycerin

Аміодарон Amiodarone

Добутамін Dobutamine

Альтеплаза Alteplase

151 / 200
Хвора 63-х років доставлена до кардіологічного відділення з приводу повної атріовентрикулярної блокади. Після призначеного лікування (нітрогліцерин, преднізолон, фуросемід, атропін, мілдронат) у неї виникла гостра затримка сечі. Який препарат привів до даного ускладнення? A 63-year-old patient was brought to the cardiology department due to complete atrioventricular block. After the prescribed treatment (nitroglycerin, prednisone, furosemide, atropine, mildronate) she developed an acute urinary retention. Which drug led to this complication?

Преднізолон Prednisone

Мілдронат Mildronate

Фуросемід Furosemide

Атропін Atropine

Нітрогліцерин Nitroglycerin

152 / 200
Літня жінка знепритомніла. Припускається, що в неї могла статися зупинка кровообігу. Час, необхідний для встановлення діагнозу клінічної смерті не повинен перевищувати: An elderly woman fainted. It is assumed that she may have had a circulatory arrest. The time required to establish a diagnosis of clinical death should not exceed:

10-15 хвилин 10-15 minutes

1-2 хвилини 1-2 minutes

10-15 секунд 10-15 seconds

30 хвилин 30 minutes

3-5 хвилин 3-5 minutes

153 / 200
Дитина 11-ти років збита автомобілем. Скарги на гострий біль у промежині та тазовій ділянці. Є підозра на перелом кісток тазу. Яка маніпуляція може зменшити біль при проведенні протишокового комплексу при переломах тазу? An 11-year-old child was hit by a car. Complaints of acute pain in the perineum and pelvic area. A fracture of the pelvic bones is suspected. What manipulation can reduce pain during anti-shock therapy complex for pelvic fractures?

Лікувальний наркоз без іммобілізації Therapeutic anesthesia without immobilization

Внутрішньом’язове введення наркотикiв Intramuscular drug administration

Внутрішньотазова блокада за Школьніковим-Селівановим Intrapelvic blockade according to Shkolnikov-Selivanov

Масивна трансфузійна терапія Massive transfusion therapy

Фіксація на щиті з випрямленими ногами Shield hold with straight legs

154 / 200
Військовослужбовець доставлений на МПП після хімічного ураження. Лікар встановив діагноз отруєння синильною кислотою. На попередніх етапах антидот не вводився. Яка перша лікарська допомога потрібна хворому? A military serviceman was brought to the hospital after a chemical injury. The doctor made a diagnosis of hydrocyanic acid poisoning. In the previous stages, no antidote was administered. What first medical aid does the patient need?

Атропін підшкірно Atropine subcutaneously

Антиціан внутрішньовенно Anticyanin intravenously

Амілнітрит інгаляційно Amyl nitrite inhalation

Унітіол внутрішньовенно Unithiol IV

Будаксим внутрішньом’язово Budaxim intramuscularly

155 / 200
До дитячого відділення був доставлений хлопчик 6-ти років з гострою гематурією. Тиждень тому переніс ГРВІ. На розгинальних поверхнях кінцівок - дрібнопапу-льозна геморагічна висипка. Помірно виражені болі в животі. Сеча темно-бурого кольору. Який найбільш імовірний діагноз? A 6-year-old boy with acute hematuria was brought to the children's department. A week ago, he suffered from SARS. On the extensor surfaces of the limbs - a small papular hemorrhagic rash. Moderate pain in the abdomen. Urine is dark brown in color. What is the most likely diagnosis?

Синдром Альпорта Alport Syndrome

Запалення сечового міхура Inflammation of the urinary bladder

Дисметаболічна нефропатія Dysmetabolic nephropathy

Гострий гломерулонефрит Acute glomerulonephritis

Геморагічний васкуліт Hemorrhagic vasculitis

156 / 200
Пацієнт 50-ти років, що знаходиться у інфарктному відділенні з трансмуральним інфарктом, раптово знепритомнів. Об’єктивно: зупинка серцевої діяльності та дихання, електромеханічна дисоціація. Розпочато серцево-легеневу реанімацію. Які наступні дії? A 50-year-old patient in the infarct department with a transmural infarction suddenly fainted. Objectively: cardiac and respiratory arrest, electromechanical dissociation. Cardiovascular -pulmonary resuscitation. What are the next actions?

Внутрішньосерцеве введення адреналіну Intracardiac administration of epinephrine

Внутрішньовенне введення лідокаїну Intravenous administration of lidocaine

Внутрішньовенне введення новокаїнаміду Intravenous administration of novocaine

Внутрішньовенне введення строфантину Intravenous Strophanthin

Дефібриляція Defibrillation

157 / 200
Жінку 77-ми років доставлено в лікарню без свідомості. Неврологічний статус: лівобічна параплегія. Протромбін за Кві-ком становить 108%. На КТ-сканах відзначається: у правій півкулі головного мозку ділянка з нерівномірним зниженням оптичної щільності. Який патологічний стан розвинувся у пацієнтки? A 77-year-old woman was brought to the hospital unconscious. Neurological status: left-sided paraplegia. Qui-com prothrombin is 108%. CT scans show: in in the right hemisphere of the brain, there is an area with an uneven decrease in optical density. What pathological condition has developed in the patient?

Субарахноїдальний крововилив Subarachnoid hemorrhage

Менінгоенцефаліт Meningoencephalitis

Геморагічний інсульт Hemorrhagic stroke

Метастази у головний мозок Metastases in the brain

Ішемічний інсульт Ischemic stroke

158 / 200
Пацієнт 42-х років госпіталізований до лікарні в невідкладному порядку. Подає скарги на безсоння, нудоту, блювання, страх. Дезорієнтований в місці та часі. Неспокійний, зривається з місця, ховається під ліжко, бачить навколо 'чорних жуків', топче їх ногами. Яке лікування є найбільш доцільне? A 42-year-old patient is admitted to the hospital urgently. Complains of insomnia, nausea, vomiting, fear. Disoriented in place and time. Restless, breaks out of places, hides under the bed, sees 'black bugs' around, stomps on them. What treatment is most appropriate?

Седуксен + детоксикація Seduxen + detoxification

Електроконвульсивна терапія Electroconvulsive therapy

Анксіолітики + психотерапія Anxiolytics + psychotherapy

Галоперидол + коректори Haloperidol + correctors

Антидепресанти + препарати літію Antidepressants + lithium drugs

159 / 200
Під час ДТП збито пішохода. При попередньому огляді бригадою ШМД у нього діагностований перелом стегна. Який з перерахованих заходів слід провести у першу чергу? During the road accident, a pedestrian was hit. During the preliminary examination by the ShMD brigade, he was diagnosed with a hip fracture. Which of the listed measures should be carried out first?

Знеболювання Pain relief

Забезпечення венозного доступу Ensure venous access

Транспортна іммобілізація Transport immobilization

Інфузійна терапія Infusion therapy

Проведення допоміжної ШВЛ Carrying out auxiliary ventilation

160 / 200
Дівчинку 13-ти років щойно знайшли у віддаленому куточку парку. Дитина збентежена, на запитання відповідати відмовляється. Одяг зіпсований. На обличчі та шиї подряпини. Свідки бачили чоловіка, що тікав з цієї частини парку. Лікар, який опинився на місці подій, має перш за все: A 13-year-old girl was just found in a remote corner of the park. The child is confused, refuses to answer questions. Her clothes are ruined. She has scratches on her face and neck. Witnesses saw a man, who was running away from this part of the park. The doctor who happened to be at the scene of the events has, first of all:

Спробувати знайти родичів дитини Try to find child's relatives

Опитати свідків Interview witnesses

Обробити подряпини на лиці та шиї Treat scratches on face and neck

Провести профілактику вагітності та венеричних хвороб Conduct prevention of pregnancy and sexually transmitted diseases

Виключити травми піхви та внутрішніх органів Exclude injuries to the vagina and internal organs

161 / 200
Вагітна 23-х років з центральним пере-длежанням плаценти розроджена в 37 тижнів гестації шляхом корпорального кесарева розтину. Операція була ускладнена гіпотонією матки, яка ліквідувалася після введення утеротоніків. Загальний об’єм крововтрати становив 900 мл. Через декілька хвилин, вже під час ушивання передньої черевної стінки, матка знову розслаблюється, тонус не відновлюється. Які подальші дії операційної бригади? A 23-year-old pregnant woman with central placenta previa delivered at 37 weeks of gestation by corporal caesarean section. The operation was complicated by hypotonia of the uterus, which was eliminated after the administration of uterotonics. The total volume of blood loss was 900 ml. After a few minutes, already during suturing of the anterior abdominal wall, the uterus relaxes again, the tone is not restored. What are the further actions of the operating team?

Посилення інфузійної терапії Intensification of infusion therapy

Ретельний облік об’єму крововтрати Careful accounting of the volume of blood loss

Хірургічна зупинка кровотечі Surgical stoppage of bleeding

Масаж матки на кулаці Massage of the uterus on the fist

Введення препарату новоссевен Introduction of drug novosseven

162 / 200
Роділля 28-ми років у пологах раптово поскаржилась на локальну болючість в дні матки. З’явилися біль за грудниною, занепокоєння, ціаноз, різка гіпотонія. Пологи завершилися операцією кесарського розтину з приводу дистресу плоду на фоні передчасного відшарування нормально розташованої плаценти. Є підозра на емболію навколоплідними водами. Об’єм операції розширено до гістеректомії з матковими трубами. Яке найбільш імовірне ускладнення слід очікувати у пацієнтки? A 28-year-old woman in labor suddenly complained of local pain in the bottom of the uterus. She developed pain behind the sternum, anxiety, cyanosis, and severe hypotension. The delivery ended with an operation cesarean section due to fetal distress on the background of premature detachment of a normally located placenta. Amniotic fluid embolism is suspected. The scope of the operation has been expanded to hysterectomy with fallopian tubes. What is the most likely complication to be expected in the patient?

Асфіксія Asphyxia

Портальна гіпєртєнзія Portal hypertension

Коагулопатія Coagulopathy

Судоми Convulsions

Аритмогенний шок Arrhythmogenic shock

163 / 200
Під час забезпечення центрального венозного доступу при транспортуванні постраждалого з місця катастрофи відмічено розвиток підшкірної емфіземи ділянки шиї та грудної клітки. Який діагноз є найбільш імовірним? During the provision of central venous access during transportation of the victim from the disaster site, the development of subcutaneous emphysema of the neck and chest area was noted. What is the most likely diagnosis?

Пункція трахеї Trache puncture

Пневмофіброз легені Pneumofibrosis of the lung

Поранення легені з розвитком пневмотораксу Lung injury with development of pneumothorax

Плевральна нориця Pleural fistula

Газова емболія легеневої артерії Gas embolism of the pulmonary artery

164 / 200
Пацієнту 66-ти років, який перебуває у відділенні інтенсивної терапії з приводу тяжкого абдомінального сепсису, для стимуляції імунних реакцій організму лікар прийняв рішення провести аутогемотрансфу-зію шляхом ін’єкції крові пацієнта внутрішньом’язово в обсязі 20 мл. Обсяг лабораторних досліджень перед даною маніпуляцією повинен включати: To a 66-year-old patient who is in the intensive care unit due to severe abdominal sepsis, the doctor decided to perform an autohemotransfusion by means of an injections of the patient's blood intramuscularly in the amount of 20 ml. The volume of laboratory tests before this manipulation should include:

При сепсисі аутогемотрансфузія протипоказана With sepsis, autohemotransfusion is contraindicated

Електроліти плазми Plasma electrolytes

ЕКГ ECG

Аналіз сечі на вільний гемоглобін Urine analysis for free hemoglobin

Лапароцентез Laparocentesis

165 / 200
У хворого 29-ти років гіперстенічної конституції, під час виконання катетеризації підключичної вени лікар діагностував пункцію артерії. Яка подальша тактика анестезіолога? In a 29-year-old patient with a hypersthenic constitution, during the catheterization of the subclavian vein, the doctor diagnosed a puncture of the artery. What are the further tactics of the anesthesiologist?

Катетеризувати артерію та проводити інфузію внутрішньоартеріально Catheterize the artery and administer intra-arterial infusion

Видалити голку, стискаюча пов’язка та холод на місце пункції Remove the needle, compression bandage and cold at the puncture site

Контрастна артеріографія для діагностики кровотечі Contrast arteriography for diagnosis of bleeding

Хірургічне ушивання дефекту артерії Surgical suturing of an artery defect

Обколоти місце пункції розчином адреналіну Surround the puncture site with adrenaline solution

166 / 200
Потерпілий 31-го року одержав однократну дозу зовнішнього опромінення в 3 Гр, доставлений на I етап медичної евакуації з вогнища радіаційного зараження зі скаргами на нудоту, блювання, постійний головний біль, слабкість, біль у кістках. Об’єктивно: свідомість ясна, збуджений. Температура тіла 37,8oC. Спостерігається ін’єкція склер, гіперемія шкірних покривів. АТ- 90/60 мм рт.ст., ЧСС- 100/хв. Який найбільш імовірний діагноз? The victim of the 31st year received a single dose of external radiation of 3 Gy, was delivered to the first stage of medical evacuation from the focus of radiation contamination with complaints of nausea, vomiting, constant headache pain, weakness, bone pain. Objectively: consciousness is clear, excited. Body temperature is 37.8oC. Scleral injection, skin hyperemia is observed. Blood pressure - 90/60 mm Hg, heart rate - 100/min. What is the most likely diagnosis?

Гостра променева хвороба, кишкова форма, період відновлення Acute radiation sickness, intestinal form, recovery period

Гостра променева хвороба, церебральна форма, період відновлення Acute radiation sickness, cerebral form, recovery period

Гостра променева хвороба, судинно-токсемічна форма, латентний період Acute radiation sickness, vascular-toxemic form, latent period

Гостра променева хвороба, кишкова форма, період розпалу захворювання Acute radiation sickness, intestinal form, period of exacerbation of the disease

Гостра променева хвороба, кістково-мозкова форма, період первинної реакції Acute radiation sickness, bone marrow form, period of primary reaction

167 / 200
Під час проведення регламентних робіт на АЕС трапилась аварія з викидом радіоактивних речовин. 32 працівника чергової зміни отримали різні дози зовнішнього опромінення. Яка доза опромінення зумовлює гостру променеву хворобу легкого ступеня? During routine work at the NPP, an accident occurred with the release of radioactive substances. 32 workers on the regular shift received various doses of external radiation. What dose of radiation causes mild acute radiation sickness ?

0,25-0,5 Гр 0.25-0.5 Gy

1-2 Гр 1-2 Gy

8-10 Гр 8-10 Gy

12-15 Гр 12-15 Gy

4-6 Гр 4-6 Gy

168 / 200
Бригадою швидкої допомоги до лікарні швидкої допомоги доставлений хворий 56-ти років, що лежав на снігу при температурі повітря —20oC. Об’єктивно: свідомість сплутана, міміка відсутня, шкіра бліда, синюшна, холодна на дотик. Ps- 52/хв., АТ-80/45 мм рт.ст., ЧД- 9-10/хв., поверхневе; різкий запах алкоголю, температура тіла 29oC. Який попередній діагноз? A 56-year-old patient was brought to the emergency hospital by the ambulance crew, who was lying on the snow at an air temperature of -20oC. Objectively: consciousness is confused, facial expressions are absent , the skin is pale, bluish, cold to the touch. Ps- 52/min., BP-80/45 mm Hg, BH- 9-10/min, superficial; sharp smell of alcohol, body temperature 29oC. What is the previous diagnosis ?

Загальне переохолодження General hypothermia

Отруєння сурогатами алкоголю Alcohol surrogate poisoning

Тяжка ступінь алкогольного сп’яніння Severe degree of alcohol intoxication

Гостре порушення мозкового кровообігу Acute cerebrovascular accident

Закрита черепно-мозкова травма Closed brain injury

169 / 200
У роділлі з надмірною пологовою діяльністю раптом з’явилися кров’янисті виділення з піхви в об’ємі 500 мл., погіршилось серцебиття плоду. Роділля зблідла, Ps-100/хв., АТ100/60 мм рт.ст. Який найбільш імовірний діагноз? In a woman in labor with excessive labor activity, vaginal bleeding in the volume of 500 ml suddenly appeared, the heartbeat of the fetus worsened. The woman in labor became pale, Ps- 100/min., BP100/60 mm Hg. What is the most likely diagnosis?

Передчасне відшарування плаценти Premature placental abruption

Прееклампсія Preeclampsia

Клінічно вузький таз Clinically narrow pelvis

Розрив шийки матки Rupture of the cervix

Розрив матки Rupture of uterus

170 / 200
Вагітна 32-х років надійшла на пологи з доношеною вагітністю. Протягом останніх 2 діб не відчуває рухи плоду. Вагітність VI, пологів III та II штучних аборти без ускладнень. Об’єктивно: загальний стан задовільний, Ps- 94/хв., температура тіла - 36,6oC, АТ- 110/70 мм рт.ст. Положення плоду поздовжнє, передлежить голівка, притиснута до входу в малий таз. Серцебиття плоду не прослуховується. Який найбільш імовірний діагноз? A 32-year-old pregnant woman went into labor with a full-term pregnancy. During the last 2 days, she does not feel the movements of the fetus. Pregnancy VI, childbirth III and II artificial abortions without complications. Objectively: the general condition is satisfactory, Ps - 94/min., body temperature - 36.6oC, BP ​​- 110/70 mm Hg. The position of the fetus is longitudinal, the head is presented, pressed against the entrance to the small pelvis. The heartbeat of the fetus is not heard . What is the most likely diagnosis?

Вагітшсть 40 тижнів. Рання неонатальна загибель плоду Pregnancy 40 weeks. Early neonatal death of the fetus

Вагітність 40 тижнів. Пізня неонатальна загибель плоду Pregnancy 40 weeks. Late neonatal death of the fetus

Вагітність 40 тижнів. Антенатальна загибель плоду Pregnancy 40 weeks. Antenatal death of the fetus

Вагітність 40 тижнів. !нтранатальна загибель плоду Pregnancy 40 weeks. !ntranatal death of the fetus

Вагітність 40 тижнів. Постнатальна загибель плоду Pregnancy 40 weeks. Postnatal death of the fetus

171 / 200
Чоловік 55-ти років раптово на вулиці знепритомнів. Пульс на сонних артеріях відсутній, дихання відсутнє, шкірні покриви ци-анотичні, зіниці розширені. Бригада швидкої допомоги прибула на місце події через 2 хвилини та успішно провела реанімаційні заходи: відновилася серцева діяльність, але самостійне дихання відсутнє. Необхідно: A 55-year-old man suddenly fainted on the street. There is no pulse on the carotid arteries, no breathing, the skin is cyanotic, the pupils are dilated. The ambulance team arrived at scene after 2 minutes and successfully carried out resuscitation measures: heart activity resumed, but there is no independent breathing. Required:

Ввести цитітон Enter city

Накласти трахеостому Apply a tracheostomy

Ввести строфантин Enter strophantine

Продовжувати ШВЛ Continue ventilator

Ввести бемегрид Enter bemegrid

172 / 200
Чоловік 82-х років протягом останніх 15-ти років хворіє на гіпертонічну хворобу, 2 роки тому переніс інфаркт міокарда. Раптово під час фізичного навантаження виник ангінозний напад, який неодноразово повторюється в спокої тривалістю 1530 хвилин. АТ-160/100 мм рт.ст., ритм серця правильний, частота 82/хв. На ЕКГ: в III і AVF відведеннях глибокий зубець Q без зміщення сегменту ST, позитивний зубець T. Який попередній діагноз? An 82-year-old man has been suffering from hypertension for the past 15 years, suffered a myocardial infarction 2 years ago. Suddenly, during physical exertion, an anginal attack occurred, which repeatedly repeated at rest for 1530 minutes. BP-160/100 mmHg, heart rhythm is correct, frequency 82/min. On ECG: in III and AVF leads deep Q wave without ST segment shift, positive T wave. What is the previous diagnosis?

Повторний інфаркт міокарда Recurrent myocardial infarction

Прогресуюча стенокардія напруження Progressive angina pectoris

Варіантна стенокардія (Принцметала) Variant angina pectoris (Prinzmetal)

Гостра розшаровуюча аневризма аорти Acute dissecting aortic aneurysm

Рання післяінфарктна стенокардія Early postinfarction angina

173 / 200
У хлопчика 14-ти років внаслідок автомобільної аварії виник перелом кісток тазу. Які умови транспортування необхідно забезпечити даному пацієнту? A 14-year-old boy suffered a pelvic fracture as a result of a car accident. What conditions of transportation should be provided for this patient?

Умови транспортування не мають значення Transport conditions are irrelevant

У положенні 'жаби'на твердій поверхні In the 'frog' position on a hard surface

У підвищеному положенні верхньої частини тулуба під кутом 30° In the elevated position of the upper body at an angle of 30°

У положенні на животі In the prone position

У сидячому положенні In a sitting position

174 / 200
У потерпілого в ДТП запідозрили перелом тіла 12-го грудного хребця. Рухи в нижніх кінцівках різко обмежені, чутливість порушена. Як слід транспортувати хворого? A fracture of the body of the 12th thoracic vertebra was suspected in the accident victim. Movements in the lower limbs are sharply limited, sensitivity is impaired. How should the patient be transported?

Лежачи на животі, підклавши під груди і голову подушку або одяг Lying on your stomach with a pillow or clothes under your chest and head

Лежачи на лівому боку Lying on the left side

На щиті On the shield

Лежачи на правому боку Lying on the right side

Лежачи на спині з іммобілізацією голови Lying on the back with head immobilization

175 / 200
При аварії на ядерному реакторі відбувся викид в навколишнє середовище значної кількості радіоізотопів йоду. Який препарат є препаратом вибору у профілактиці ураження щитоподібної залози? In the event of a nuclear reactor accident, a significant amount of radioisotopes of iodine was released into the environment. Which drug is the drug of choice in the prevention of thyroid damage?

Йодомарин Iodomarin

5% спиртовий розчин йоду 5% alcohol solution of iodine

Антиструмін Anticurrent

Розчин Люголю Lugol's solution

Йодид калію Potassium iodide

176 / 200
На ядерному реакторі сталася позаштатна ситуація. До навколишнього середовища потрапила значна кількості радіоізотопів йоду. Яка тривалість прийому препаратів стабільного йоду для профілактики ураження щитоподібної залози? An extraordinary situation occurred at the nuclear reactor. A significant amount of iodine radioisotopes got into the environment. What is the duration of taking stable iodine preparations for the prevention of thyroid damage?

10 днів 10 days

14 днів 14 days

До зникнення загрози потрапляння в організм радіоактивних ізотопів йоду Until the disappearance of the threat of radioactive iodine isotopes entering the body

7 днів 7 days

30 днів 30 days

177 / 200
Хвора 44-х років впала на ліву витягнуту руку. При обстеженні виявлено набряк та болючість при пальпації в ділянці лівого променево-зап’ясного суглоба, крепітація. Лікар швидкої допомоги вирішив провести транспортну іммобілізацію шиною Краме-ра. Який вид пов’язки застосовується для фіксації шини? A 44-year-old female patient fell on her left outstretched arm. During the examination, swelling and tenderness on palpation in the area of ​​the left radio-carpal joint, crepitation. Emergency doctor help decided to carry out transport immobilization with Kramer's splint. What type of bandage is used to fix the splint?

Дезо Deso

Спіральна Spiral

Вельпо Welpo

Дельбе Delbe

Косинкова Kosynkova

178 / 200
Пішохід 26-ти років, збитий автомашиною, вдарився головою при падінні об бордюр тротуару. Стан середньої важкості. Є загальна загальмованість при збереженій свідомості та орієнтуванні, гіперемія обличчя, локалізований головний біль на боці удару. Хворий лежить на боці удару. Відмічаються брадикардія, підвищення артеріального тиску, зниження м’язової сили а також різниця в реакції зіниць на світло. Як транспортувати потерпілого? A 26-year-old pedestrian, hit by a car, hit his head when he fell on the curb of the sidewalk. The condition is of medium severity. There is general retardation with preserved consciousness and orientation, facial hyperemia, localized headache on the side of the impact. The patient lies on the side of the impact. Bradycardia, increased blood pressure, decreased muscle strength, and a difference in the reaction of the pupils to light are noted. How to transport the victim?

Голова на хворому боці, фіксована до шини Єланського з піднятим головним кінцем Head on the affected side, fixed to the Yelansky splint with the head end raised

Голова на здоровому боці, фіксована до шини Єланського з піднятим головним кінцем The head is on the healthy side, fixed to the Yelansky splint with the head end raised

Голова на здоровому боці, фіксована до шини Єланського The head is on the healthy side, fixed to the Yelansky splint

Голова на хворому боці, фіксована до шини Єланського Head on the affected side, fixed to the Yelansky splint

Іммобілізація голови забезпечується за допомогою шини Крамера Immobilization of the head is provided with the help of a Kramer splint

179 / 200
Пішохід 52-х років під час ожеледиці послизнувся та впав на бордюр тротуару грудною кліткою. Скаржиться на локалізований біль у ділянці грудної клітки справа по задній пахвовій лінії. Відмічаються тахікардія, підвищення артеріального тиску, припухлість, синець та крепітація в ділянці 9-10 ребра справа. При диханні та кашлі -різкий біль. Запідозрено перелом ребер. Як слід транспортувати хворого? A 52-year-old pedestrian slipped and fell on the curb of the sidewalk with his chest during the ice. He complains of localized pain in the right chest area along the posterior axillary line. Tachycardia is noted , increased blood pressure, swelling, bruising and crepitus in the area of ​​the 9-10th rib on the right. Sharp pain when breathing and coughing. A rib fracture is suspected. How should the patient be transported?

В напівсидячому положенні, наклавши окклюзійну пов’язку In a semi-sitting position, wearing an occlusive bandage

В напівсидячому положенні з палкою в ліктьових згинах In a semi-sitting position with a stick in the elbows

Наклавши шину Крамера з фіксацією плечового, ліктьового, променевозап’ясткового суглобів Having applied Kramer's splint with fixation of shoulder, elbow, radiocarpal joints

Наклавши пов’язку Дезо з валиком в пахвовій ділянці Having applied a Deso bandage with a roller in the armpit

Прибинтувавши праву верхню кінцівку до грудної клітки Bandaging the right upper limb to the chest

180 / 200
Чоловік 49-ти років знайдений після ав-тоаварії в тяжкому стані. При огляді міжреберні проміжки згладжені, ліва половина грудної клітки відстає при диханні. Над лівим легеневим полем тимпаніт, аускульта-тивно - дихання зліва не проводиться, ЧД-32/хв. Інші пошкодження відсутні. Як транспортувати потерпілого? A 49-year-old man was found in a serious condition after a car accident. On examination, the intercostal spaces are smoothed, the left half of the chest lags behind during breathing. Tympanitis over the left lung field , by auscultation - breathing is not performed on the left side, CH-32/min. There are no other injuries. How to transport the victim?

На правому боці лежачи Lying on the right side

В напівсидячому чи напівлежачому положенні In a semi-sitting or semi-lying position

На лівому боці лежачи Left side lying

В положенні Волковича на щиті In the position of Volkovich on the shield

Грудна клітка фіксується шинами Краме-ра до щита The chest is fixed with Kramer's tires to the shield

181 / 200
Хворий 24-х років 4 години тому одержав травму правого плеча, упавши на лікоть. Стан середньої тяжкості, Ps-100/хв., АТ-100/60 мм рт.ст. Праве плече нижче плечового суглоба деформоване, рухи в ньому неможливі, виражена болісність. Яка іммобілізація кінцівки для транспортування хворого до травматологічного відділення? A 24-year-old patient injured his right shoulder 4 hours ago, falling on his elbow. Moderate condition, Ps-100/min., BP-100/60 mm Hg. The right shoulder below the shoulder joint is deformed, movements in it are impossible, there is severe pain. What is the immobilization of the limb for transporting the patient to the trauma department?

Використання шини Дітерихса Using Dieterichs bus

Використання шини Крамера Using Kramer Bus

Кісткове витяжіння Bone extraction

Позавогнищевий остеосинтез Extrafocal osteosynthesis

Косиночна іммобілізація Mouse immobilization

182 / 200
Жінка 29-ти років при вагітності 11-12 тижнів поступила в гінекологічне відділення для штучного аборту. Вагітність 8-а, із них 2 закінчились пологами, 5 - штучними абортами ( останній аборт - 7 місяців тому). Під час операції діагностовано перфорацію матки. Подальша тактика ведення хворої? A 29-year-old woman, 11-12 weeks pregnant, was admitted to the gynecological department for an artificial abortion. She was 8th pregnant, 2 of them ended in childbirth, 5 - artificial abortions (the last abortion was 7 months ago). During the operation, a perforation of the uterus was diagnosed. Further tactics of managing the patient?

Припинити виконання аборту, спостерігати за станом жінки Stop the abortion, monitor the woman's condition

Припинити виконання аборту, виписати жінку під нагляд жіночої консультації Stop the abortion, discharge the woman under the supervision of a women's consultation

Закінчити штучний аборт, призначити антибактеріальні засоби Finish artificial abortion, prescribe antibacterial agents

Припинити виконання аборту, термінова лапаротомія Stop abortion, emergency laparotomy

Закінчити штучний аборт, призначити антигеморагічні засоби End artificial abortion, prescribe antihemorrhagic drugs

183 / 200
Робітник АЄС змушений тимчасово залишатися на забрудненій радіоактивними речовинами місцевості. Які заходи необхідно йому вжити? A nuclear power plant worker is forced to temporarily stay in an area contaminated with radioactive substances. What measures should he take?

Не виконувати жодних дій до проведення масової евакуації Do not take any action until mass evacuation

Негайно одягнути респіратор та провітрити приміщення Immediately wear a respirator and ventilate the room

Одягнути чоботи, рукавички та комбінезон Put on boots, gloves and overalls

Самостійно виконати евакуацію Evacuate yourself

Герметизація приміщення та дезактивація внутрішніх поверхонь Sealing of the room and decontamination of internal surfaces

184 / 200
Група військовослужбовців тривалий час знаходилась у кімнаті під впливом іонізуючого випромінювання після аварії на АЕС. Які першочергові заходи потрібно застосувати для зменшення впливу іонізуючого випромінювання? A group of military personnel was in a room exposed to ionizing radiation for a long time after an accident at a nuclear power plant. What primary measures should be taken to reduce exposure to ionizing radiation?

Надіти протигаз Put on a gas mask

Провітрити приміщення Ventilate the room

Прийняти демиткарб Accept demitkarb

Перевдягти верхній одяг Change outer clothes

Не вживати жодних дій Take no action

185 / 200
Група постраждалих була евакуйована із зони застосування хімічної зброї. Відомо, що усі наділи протигази, але у двох з групи відмічались постійне моторне збудження, вони щось нерозбірливо бормочуть, зіниці звужені, фібриляція, посмикування м’язів обличчя, ЧД32 /хв, видих утруднений. Який вид хімічної зброї був застосований? A group of victims was evacuated from the area where chemical weapons were used. It is known that everyone put on gas masks, but two of the group had constant motor excitement, they are mumbling something unintelligibly, pupils narrowed, fibrillation, twitching of facial muscles, BH32/min, exhalation is difficult. What type of chemical weapon was used?

Подразливої та сльозоточивої дії Irritable and lachrymatory action

Нервово-паралітичної дії Nerve-paralytic effect

Шкірнонаривної дії Skin-abrasive action

Загальної дії General

Задушливої дії Suffocating action

186 / 200
Хвора 37-ми років прооперована з приводу фіброміоми матки. Виконана екстирпація матки. Через 3 доби після операції хвора скаржиться на біль у ділянці правого стегна, підвищення температури до 38, 5oC, набряк нижньої кінцівки. Об’єктивно: права нижня кінцівка збільшена у діаметрі, шкірні покрови теплі на дотик, синюшного кольору. Встановлено діагноз: ілеофомо-ральний тромбоз правої нижньої кінцівки. При ретроградній ілеокаваграфії виявлено флотуючий тромб. Яка лікувальна тактика? A 37-year-old patient was operated on for uterine fibroids. The uterus was extirpated. 3 days after the operation, the patient complains of pain in the right thigh area, a temperature rise to 38 ...

Еластична компресія Elastic Compression

Консервативне медикаментозне лікування Conservative medical treatment

Лікування не потребує Does not require treatment

Фізіотерапевтичне лікування Physiotherapy treatment

Оперативне лікування Operative treatment

187 / 200
Хвору 69-ти років доставлено до приймального відділення з діагнозом гостре порушення мозкового кровообігу в лівій гемісфері, правостороння геміплегія. Хвора контактна, але дезорієнтована, спонтанно розплющує очі, виконує інструкції. Який рівень свідомості за шкалою ком Глазго? A 69-year-old patient was brought to the admissions department with a diagnosis of acute cerebral circulation disorder in the left hemisphere, right-sided hemiplegia. The patient is in contact, but disoriented, spontaneously opens her eyes, performs instructions. What is the level of consciousness according to the Glasgow coma scale?

Смерть мозку (3 бали) Brain death (3 points)

Сопор (9-12 балів) Sopor (9-12 points)

Ясна свідомість (15 балiв) Clear consciousness (15 points)

Оглушення (13-14 балів) Stun (13-14 points)

Кома (4-8 балів) Comma (4-8 points)

188 / 200
Під час огляду дівчинка 8-ми років раптово знепритомніла. Шкіра та слизові блідоціанотичні. Екскурсій грудної клітки немає. Пульс на магістральних артеріях не визначається. Який найбільш імовірний діагноз? During the examination, an 8-year-old girl suddenly fainted. The skin and mucous membranes are pale cyanotic. There are no chest excursions. The pulse on the main arteries is not detected. What is the most likely diagnosis?

Зомління Grinding

Кома Comma

Набряк легенів Pulmonary edema

Колапс Collapse

Стан клінічної смерті State of clinical death

189 / 200
Хворий 47-ми років, що постраждав унаслідок ДТП, скаржиться на різкий біль у правому стегні, психомоторне збудження. Об’єктивно: шкіра бліда, волога, позитивний симптом 'білої'плями. АТ- 80/40 мм рт.ст., ЧСС- 120/хв. Локально виявляється деформація верхньої третини правого стегна та його патологічна рухливість. Чим зумовлена важкість стану постраждалого? A 47-year-old patient who was injured in a road accident complains of sharp pain in the right thigh, psychomotor agitation. Objectively: the skin is pale, moist, a positive symptom 'white' spot. Blood pressure - 80/40 mm Hg, heart rate - 120/min. Locally, the deformation of the upper third of the right thigh and its pathological mobility are detected. What is the cause of the severity of the victim's condition?

Травматичний шок IV ступеня Traumatic shock IV degree

Больовий синдром Pain syndrome

Травматичний шок I ступеня Traumatic shock of the first degree

Травматичний шок II ступеня Traumatic shock II degree

Травматичний шок III ступеня Traumatic shock III degree

190 / 200
Триває значний викид радіонуклідів у атмосферу з пошкодженого ядерного реактора. Доза опромінення всього тіла за перші 10 діб у дорослих становить 500 мЗв. Які заходи захисту треба запровадити? A significant release of radionuclides into the atmosphere from a damaged nuclear reactor continues. The dose of radiation to the whole body for the first 10 days in adults is 500 mSv. What protective measures should be implemented?

Потрібно провести йодну профілактику It is necessary to carry out iodine prophylaxis

Ніякі, тому що дози опромінення не перевищують допустимі рівні None, because radiation doses do not exceed permissible levels

Використовуються укриття та засоби захисту органів дихання та шкірного покрову Shelter and means of respiratory and skin protection are used

За таких умов потрібна евакуація населення із зон радіоактивного забруднення Under such conditions, population evacuation from radioactive contamination zones is required

Обмежити вживання забруднених радіонуклідами харчових продуктів та питної води Limit consumption of food and drinking water contaminated with radionuclides

191 / 200
Лейтенант отримав ураження у вогнищі застосування супротивником хімічної зброї. Одночасно був поранений уламком міни в праве стегно. Через 2-3 години у потерпілого виник біль у рані, що наростав, свербіж та відчуття печіння. Пов’язка на рані видає інтенсивний часниковий запах. Шкіра навколо рани покрита краплями маслянистої рідини чорного кольору, набрякла, гіперемована, з множинними міхурами жовтого кольору. Яку хімічну речовину найімовірніше застосували на полі бою? The lieutenant was injured in the focus of the enemy's use of chemical weapons. At the same time, he was wounded by a fragment of a mine in the right thigh. After 2-3 hours, the victim developed pain in the wound, which grew , itching and a burning sensation. The bandage on the wound emits an intense garlic smell. The skin around the wound is covered with drops of an oily black liquid, swollen, hyperemic, with multiple yellow blisters. What chemical substance was most likely used on the battlefield?

Синильна кислота Pranic acid

Зарін Zarin

Люїзит Lewisite

Зоман Call

!прит !pret

192 / 200
В момент повітряного ядерного вибуху військовослужбовець отримав світлове опромінення. При цьому відчув короткочасну втрату зору, набряклість повік, відчуття обпечення шкіри обличчя та обох кистей. Через дві години після вибуху відчув слабкість, запаморочення, з’явилась нудота, було одноразове блювання. Об’єктивно: шкіра обличчя, тилу кистей сильно гіперемова-ні, помірно набряклі. Зір збережений, повіки набряклі, очні щілини звужені, кон’юнктива гіперемована, відзначається сльозотеча. Показання індивідуального дозиметру - 2,5 Гр. Який найбільш імовірний діагноз? At the time of the aerial nuclear explosion, the serviceman received light exposure. At the same time, he experienced a short-term loss of vision, swelling of the eyelids, a burning sensation on the skin of the face and both hands. Two hours after the explosion felt weak, dizzy, nausea appeared, there was a single vomiting. Objectively: the skin of the face, the back of the hands is very hyperemic, moderately swollen. Vision is preserved, the eyelids are swollen, the eye slits are narrowed, the conjunctiva is hyperemic, lacrimation is noted. Indications of an individual dosimeter - 2.5 Gy. What is the most probable diagnosis?

Комбіноване радіаційне ураження середньої тяжкості. Термічний опік світловим випроміненням обличчя, обох очей, обох кистей рук 7% III ступеня. Променева хвороба II ступеня, гострий період Combined radiation damage of medium severity. Thermal burn with light radiation of the face, both eyes, both hands 7% III degree. Radiation sickness II degree, acute period

Радіаційне ураження середньої тяжкості. Термічний опік світловим випроміненням обличчя, обох очей, обох кистей рук 7% II ступеня. Променева хвороба I ступеня, гострий період Radiation damage of medium severity. Thermal burn with light radiation of the face, both eyes, both hands 7% II degree. Radiation sickness I degree, acute period

Комбіноване радіаційне ураження середньої тяжкості. Термічний опік обличчя, обох очей, обох кистей рук 7% I ступеня. Променева хвороба I ступеня, гострий період Combined radiation damage of medium severity. Thermal burn of the face, both eyes, both hands 7% of the 1st degree. Radiation sickness of the 1st degree, acute period

Комбіноване радіаційне ураження середньої тяжкості. Термічний опік світловим випроміненням обличчя, обох очей, обох кистей рук 7% I ступеня. Променева хвороба I ступеня, гострий період Combined radiation damage of medium severity. Thermal burn with light radiation of the face, both eyes, both hands 7% of the 1st degree. Radiation sickness of the 1st degree, acute period

Комбіноване радіаційне ураження середньої тяжкості. Термічний опік світловим випроміненням обличчя, обох очей, обох кистей рук 7% I ступеня. Променева хвороба II ступеня, гострий період Combined radiation injury of medium severity. Thermal burn with light radiation of the face, both eyes, both hands 7% I degree. Radiation sickness II degree, acute period

193 / 200
Офіцер 23-х років отримав осколкове поранення правої верхньої кінцівки. Стан задовільний. Ps- 82/хв., АТ- 110/70 мм рт.ст. На передній поверхні правого плеча у середній третині визначається рана розмірами 1,0х0,5 см. Пульсація на променевій артерії збережена. Виявляється 'звисаюча кисть', відсутність активного розгинання кисті та основних фаланг пальців, супінації кисті. Неможливо відведення великого пальця. Відмічається також порушення чутливості на тильній поверхні передпліччя, променевій половині тилу кисті та у ділянці першого міжпальцевого проміжку. Який нерв пошкоджено? A 23-year-old officer received a shrapnel injury to the right upper limb. His condition is satisfactory. Ps- 82/min., BP- 110/70 mm Hg. On a wound measuring 1.0x0.5 cm is determined on the front surface of the right shoulder in the middle third. Pulsation on the radial artery is preserved. There is a 'hanging hand', the absence of active extension of the hand and the main phalanges of the fingers, supination of the hand. It is not possible to abduct the thumb. There is also a violation of sensitivity on the back surface of the forearm, the radial half of the back of the hand and in the area of ​​the first interdigital space. What nerve is damaged?

Ліктьовий нерв Ulnar nerve

Серединний нерв Median nerve

Променевий нерв Radiant nerve

Ліктьовий та серединний нерви Ulnar and median nerves

Ліктьовий та променевий нерви Ulnar and radial nerves

194 / 200
Доярка 24-х років, що мала контакт з хворими тваринами, захворіла гостро з ознобу, гарячки, болю голови та в м’язах. На 2-й день приєдналися відчуття печіння в роті, слинотеча, кон’юнктивіт, печіння під час сечовиділення. На шкірі кистей, видимих слизових оболонках - дрібні міхурці, ерозії й виразки на тлі гіперемії та набряку. Який найбільш імовірний діагноз? A 24-year-old milkmaid who had contact with sick animals became acutely ill with chills, fever, headache and muscle pain. On the 2nd day a burning sensation in the mouth, drooling, conjunctivitis, burning during urination joined. On the skin of the hands, visible mucous membranes - small blisters, erosions and ulcers against the background of hyperemia and edema. What is the most likely diagnosis?

Хвороба Бехчета Behcet's disease

Ящур FMD

Афтозний стоматит Aphthous stomatitis

Іерпетична ангіна Herpetic sore throat

Вітряна віспа Chicken Pox

195 / 200
У хворого 27-ми років на стегні виникло почервоніння і невелика болісність у пахвинній ділянці. Об’єктивно: округле вогнище гіперемії шкіри з просвітленням у центрі, помірно болісний регіонарний лімфовузол у паху. Температура нормальна. Близько тижня тому з шкіри ноги зняв кліща. Який найбільш імовірний діагноз? A 27-year-old patient developed redness on the thigh and slight pain in the inguinal area. Objectively: a round focus of skin hyperemia with a light in the center, moderately painful regional a lymph node in the groin. The temperature is normal. About a week ago, he removed a tick from the skin of his leg. What is the most likely diagnosis?

Бешиха Beshikha

Епідемічний бореліоз Epidemic borreliosis

Лімфоаденіт Lymphoadenitis

Кліщовий системний бореліоз Tick-borne systemic borreliosis

Еризипелоїд Erysipeloid

196 / 200
Захворювання розпочалося гостро. Наявна двохвильова гарячка, біль у крижах і куприку, продромальний висип (роси) у трикутнику Сімонса, етапність і мономор-фність висипу з наступним нагноєнням. Який попередній діагноз? The disease began acutely. There is a two-wave fever, pain in the sacrum and coccyx, a prodromal rash (dews) in Simon's triangle, the stage and monomorphism of the rash followed by suppuration. What is the previous diagnosis?

Мавпяча віспа Monkeypox

Синдром Стівенса-Джонсона Stevens-Johnson syndrome

Натуральна віспа Natural smallpox

Іеморагічний діатез Hemorrhagic diathesis

Вітряна віспа Chicken Pox

197 / 200
Хворий 23-х років, житель Криму, захворів гостро: температура 40oC, болі в животі та попереку. Стан тяжкий, геморагічний висип на животі. Діурез знижений, макрогематурія. Яка найбільш імовірна патологія, що зумовлює таку картину? A 23-year-old patient, a resident of Crimea, became acutely ill: temperature 40oC, pain in the abdomen and lower back. The condition is severe, hemorrhagic rash on the abdomen. Decreased diuresis, macrohematuria . What is the most likely pathology causing such a picture?

Лептоспіроз Leptospirosis

Висипний тиф Typhoid

Бубонна форма туляремії Bubonic form of tularemia

Іемоглобінурійна лихоманка Hemoglobinuria fever

Кримська геморагічна лихоманка Crimean hemorrhagic fever

198 / 200
У хворого 41-го року, який повернувся із Перу, протягом 3-х діб температура до 39oC, сильний головний біль, міальгії, нудота. Об’єктивно: шкіра обличчя та верхньої половини тулуба багряного кольору, 'кролячі очі', одутлість язика, губ. Збільшення та болючість печінки, носова кровотеча. Запідозрена жовта лихоманка. Які протиепідемічні заходи необхідно провести у родині? A 41-year-old patient who returned from Peru had a temperature of up to 39oC, severe headache, myalgia, nausea for 3 days. Objectively: the skin of the face and the upper half of the body is purple in color, 'rabbit eyes', swelling of the tongue, lips. Enlargement and tenderness of the liver, nosebleeds. Yellow fever is suspected. What anti-epidemic measures should be taken in the family?

Карантин на 10 днів Quarantine for 10 days

Екстрена вакцинація Emergency vaccination

Нічого не проводити Do nothing

Екстрена хіміопрофілактика Emergency chemoprophylaxis

Екстрена госпіталізація Emergency hospitalization

199 / 200
Жінці 32-х років з приводу пневмонії був призначений ампіцилін. Через 30 хвилин після внутрішньом’язової ін’єкції препарату хвора відчула різку слабкість, свербіж шкіри обличчя та рук, нудоту; з’явилися кашель, задишка, біль у грудній клітці. Об’єктивно: ціаноз, набряк повік та обличчя з червоними висипаннями. Ps- 120/хв, АТ-70/20 мм рт.ст. Тони серця глухі, дихання часте, поверхневе, з різнокаліберними сухими хрипами. Варикозне розширення вен на правій гомілці. Яка найбільш вірогідна причина раптового погіршення стану хворої? A 32-year-old woman was prescribed ampicillin for pneumonia. 30 minutes after the intramuscular injection of the drug, the patient felt sharp weakness, itching of the skin of the face and hands , nausea; cough, shortness of breath, chest pain appeared. Objectively: cyanosis, swelling of the eyelids and face with red rashes. Ps- 120/min, BP-70/20 mm Hg. Heart sounds dull, breathing frequent, superficial, with dry rales of various calibers. Varicose veins on the right lower leg. What is the most likely reason for the sudden deterioration of the patient's condition?

Анафілактичний шок Anaphylactic shock

Набряк Квінке Quincke edema

Тромбоемболія гілок легеневої артерії Thromboembolism of the branches of the pulmonary artery

Астматичний напад Asthma attack

Кропивниця Hives

200 / 200
У чоловіка 26-ти років на фоні бронхопневмонії розвинулася кома. Об’єктивно: ЧСС122 /хв, екстрасистолія. АТ- 80/45 мм рт.ст. Шкіра суха, тургор знижений. Дихання глибоке, шумне, рідке. Різкий запах ацетону. Печінка +5 см. Глікемія - 32 ммоль/л.; рН крові - 7,0. Який розчин буде найбільш ефективним для нормалізації дихання? A 26-year-old man developed a coma against the background of bronchopneumonia. Objectively: heart rate 122/min, extrasystole. Blood pressure - 80/45 mmHg. Skin dry, low turgor. Breathing is deep, noisy, thin. Sharp smell of acetone. Liver +5 cm. Glycemia - 32 mmol/l.; blood pH - 7.0. Which solution will be most effective for normalizing breathing?

0,9% розчин натрію хлориду 0.9% sodium chloride solution

4,2% розчин натрію бікарбонату 4.2% sodium bicarbonate solution

1% розчин калію хлориду 1% potassium chloride solution

5% розчин глюкози 5% glucose solution

Реополіглюкін Rheopoliglyukin